Как составить уровнение в задаче

Методика по просьбам трудящихся.

Я уже писал о “вопросной” системе решения текстовых задач. Суть её в том, что для решения задачи нужно выписывать вопросы, которые начинаются со слова “сколько…?”, вне зависимости от того, знаем мы ответ на них или нет. Каждый такой вопрос образует одну величину в задаче, поэтому для более старших детей можно заменять некоторые вопросы на названия величин. Например вопрос “сколько километров проезжает машина за каждый час?” неплохо заменяется на величину “скорость машины”.

Эта система оказывается настолько мощной, что позволяет решить практически любую текстовую задачу. В том числе, задачи, требующие составления уравнений.

В качестве примера и основы статьи я возьму классическую задачу для третьего класса про велосипеды.

"Что брать за X?" Как составлять уравнения к задачам

В Детском Мире продавали двухколесные трехколесные велосипеды. Максим пересчитал все велосипеды и все колеса, получилось 12 велосипедов и 27 колес. Сколько трехколесных велосипедов продавали в Детском мире?

Составим список вопросов для этой ситуации. Кроме того, на каждый вопрос постараемся дать ответ. Я не буду сейчас увлекаться объяснением, как из текста извлечь значения величин, и как понять, какие вопросы должно задавать (это слишком долго).

У меня получился такой список.

"Что брать за X?" Как составлять уравнения к задачам

Остались 4 пустые ячейки, в которые мы не можем вписать ответы – их нет в тексте, и нельзя вычислить из имеющихся.

В этом месте начинается подбор. Он в таких задачах есть всегда, просто иногда это неявный подбор в виде уравнения. Учителю в этой ситуации рекомендуется задать вопрос: “а если бы ты знал ответ на один из этих вопросов, смог бы ответить на все остальные?”

Обычно ученик отвечает утвердительно и даже указывает на вопрос, но если нет, то можно и натолкнуть – предложить свой “например”.

Первый раз подбор всегда надо осуществлять явно, поэтому учитель предлагает написать карандашом в качестве ответа “любое” число. Важно следить, чтобы это число на совпало с ответом (такое часто бывает при переучивании старшеклассников, поэтому для них нужно подбирать задачи позаковыристее, но не в плане сложного уравнения, а такие, чтобы решить было легко, но чтобы или выбор “икса” был неочевиден, или числа “напрашивались” бы неправильные).

И исходя из этого “карандашного” числа вычислить ответы на все остальные вопросы.

Так как учитель проследил, что число неправильное, в какой-то момент ребенок столкнется с противоречием в задаче. Не всегда это противоречие видно сразу, иногда приходится на него намекнуть (поэтому для первых раз нужно выбирать задачи с легко обнаруживаемыми противоречиями).

Сейчас на примере покажу, и будет ясно, что я имею в виду.

Скажем, ребенок решил, что ему поможет ответ на вопрос номер 2 – “сколько двухколёсных велосипедов?”. Он отвечает на него, допустим, числом 3. Тогда он легко вычисляет, что трёхколёсных велосипедов будет 12-3=9, колёс у двухколёсных будет 3*2=6 штук, колёс у трёхколёсных будет 9*3=27 штук. Эти все вычисления записываются в список карандашом:

"Что брать за X?" Как составлять уравнения к задачам

Я специально выбрал в качестве нулевого приближения число 3, чтобы противоречие было очевидным: если мы сложим все получившиеся колёса, то будет явно больше 27. Но ребёнок может и не заметить этого сразу. Можно сказать, что противоречие есть и предложить поискать его – довольно быстро найдёт. А то и прямо показать.

Карандашные числа стираются, и пишутся по новой. Через пару итераций выясняется, что противоречия нет, только если в качестве ответа на 2й вопрос взять число 9 (12-9=3, 9*2=18, 3*3=9, 18+9=27)

В принципе, задача решена, на этом надо бы заканчивать. Но мы разбираем решение задач через уравнения. Поэтому.

Что брать за икс?

За “икс” берём то число, которое мы подбирали (а не то, что спрашивается в задаче). И после этого ответ на каждый вопрос записываем выражением:

"Что брать за X?" Как составлять уравнения к задачам

Это оказывается довольно легко для 7-8 класса, а в младших и не требуется.

А вот что трудно – это из этих выражений составить уравнение. Можно спросить ученика “как ты понял, что три не подходит, а девять подойдёт?” Ну, и если уж не может сказать, то помочь – если брать три, то сумма ответов на 6й и 7й вопросы не будет равна 27, а если взять 9, то сумма ответов на 6й и 7й вопросы будет равна 27. Тут хотя бы у одного из собеседников предполагается умение записывать фразу математической записью.

x*2+(12-x)*3=27

То есть, вот это противоречие, которое мы искали вначале, оно же является критерием подбора, и оно же является фактом, на который составляется уравнение. Это и есть ключевой момент. И пока ученик не может видеть эти противоречия – у него не получится составить уравнение к задаче иначе, как по ключевым словам.

Решение задачи обычно свóдится к тому, чтобы путем логических рассуждений и вычислений найти значение какой-нибудь величины. Например, найти скорость, время, расстояние, массу какого-нибудь предмета или количество чего-то.

Такую задачу можно решить с помощью уравнения. Для этого искомое значение обозначают через переменную, затем путем логических рассуждений составляют и решают уравнение. Решив уравнение, производят проверку на то, удовлетворяет ли решение уравнения условиям задачи.

Запись выражений, содержащих неизвестное

Решение задачи сопровождается составлением уравнения к этой задаче. На начальном этапе изучения задач желательно научиться составлять буквенные выражения, описывающие ту или иную жизненную ситуацию. Этот этап не является сложным и его можно изучать в процессе решения самой задачи.

Рассмотрим несколько ситуаций, которые можно записать с помощью математического выражения.

Задача 1. Возраст отца x лет. Мама на два года младше. Сын младше отца в 3 раза. Запишите возраст каждого с помощью выражений.

Решение:

возраст отца мамы и сына таблица 1


Задача 2. Возраст отца x лет, мама на 2 года младше отца. Сын младше отца в 3 раза, дочь младше матери в 3 раза. Запишите возраст каждого с помощью выражений.

Решение:

возраст отца мамы сына и дочери таблица 2


Задача 3. Возраст отца x лет, мама на 3 года младше отца. Сын младше отца в 3 раза, дочь младше матери в 3 раза. Сколько лет каждому, если общий возраст отца, мамы, сына и дочери составляет 92 года?

Решение:

В данной задаче помимо записи выражений, необходимо вычислить возраст каждого члена семьи.

Сначала запишем возраст каждого члена семьи с помощью выражений. За переменную x примем возраст отца, и далее пользуясь этой переменной составим остальные выражения:

возраст отца мамы сына и дочери таблица 3

Теперь определим возраст каждого члена семьи. Для этого нам нужно составить и решить уравнение. Все компоненты уравнения у нас уже готовы. Осталось только собрать их воедино.

Общий возраст в 92 года получился путем сложения возрастов папы, мамы, сына и дочери:

возраст отца мамы сына и дочери рисунок 1

Для каждого возраста мы составили математическое выражение. Эти выражения и будут компонентами нашего уравнения. Давайте соберем наше уравнение согласно данной схеме и таблице, которая была приведена выше. То есть слова папа, мама, сын, дочь заменим на соответствующее им в таблице выражение:

возраст отца мамы сына и дочери уравнение 1

Выражение, отвечающее за возраст мамы x − 3, для наглядности было взято в скобки.

Теперь решим получившееся уравнение. Для начала можно раскрыть скобки там, где это можно:

возраст отца мамы сына и дочери уравнение 2

Чтобы освободить уравнение от дробей, умножим обе части на 3

возраст отца мамы сына и дочери уравнение 3

Решим получившееся уравнение, пользуясь известными тождественными преобразованиями:

возраст отца мамы сына и дочери уравнение 4

Мы нашли значение переменной x. Эта переменная отвечала за возраст отца. Значит возраст отца составляет 36 лет.

Зная возраст отца, можно вычислить возрасты остальных членов семьи. Для этого нужно подставить значение переменной x в те выражения, которые отвечают за возраст конкретного члена семьи.

В задаче было сказано, что мама на 3 года младше отца. Ее возраст мы обозначили через выражение x−3. Значение переменной x теперь известно, и чтобы вычислить возраст мамы, нужно в выражении x − 3 вместо x подставить найденное значение 36

x − 3 = 36 − 3 = 33 года маме.

Аналогично определяется возраст остальных членов семьи:

возраст отца мамы сына и дочери таблица 4

возраст отца мамы сына и дочери уравнение 5

Проверка:

возраст отца мамы сына и дочери проверка 1


Задача 4. Килограмм яблок стоит x рублей. Запишите выражение, вычисляющее сколько килограмм яблок можно купить на 300 рублей.

Решение

Если килограмм яблок стоит x рублей, то на 300 рублей можно купить 300 na x килограмм яблок.

Пример. Килограмм яблок стоит 50 рублей. Тогда на 300 рублей можно купить 300 na 50, то есть 6 килограмм яблок.


Задача 5. На x рублей было куплено 5 кг яблок. Запишите выражение, вычисляющее сколько рублей стоит один килограмм яблок.

Решение

Если за 5 кг яблок было уплачено x рублей, то один килограмм будет стоит x na 5 рублей

Пример. За 300 рублей было куплено 5 кг яблок. Тогда один килограмм яблок будет стоит 300 na 5, то есть 60 рублей.


Задача 6. Том, Джон и Лео на перемене пошли в столовую и купили по бутерброду и по кружке кофе. Бутерброд стоит x рублей, а кружка кофе — 15 рублей. Определите стоимость бутерброда, если известно, что за всё было уплачено 120 рублей?

Решение

Конечно, данная задача проста как три копейки и ее можно решить не прибегая к уравнению. Для этого из 120 рублей нужно вычесть стоимость трех кружек кофе (15 × 3), и полученный результат разделить на 3

стоимость бутерброжов и кофе простое решение

Но наша цель — составить уравнение к задаче и решить это уравнение. Итак, стоимость бутерброда x рублей. Куплено их всего три. Значит увеличив стоимость в три раза, мы получим выражение описывающее сколько рублей было уплачено за три бутерброда

3x — стоимость трех бутербродов

А стоимость трех кружек кофе можно записать как 15 × 3. 15 это стоимость одной кружки кофе, а 3 множитель (Том,  Джон и Лео), увеличивающий эту стоимость в три раза.

По условию задачи за все уплачено 120 рублей. У нас уже появляется примерная схема, что нужно делать:

стоимость бутербродов и кофе схема

Выражения, описывающие стоимость трех бутербродов и трех кружек кофе, у нас уже готовы. Это выражения 3x и 15 × 3. Пользуясь схемой составим уравнение и решим его:

стоимость бутерброжов и кофе решение к задаче

Итак, стоимость одного бутерброда составляет 25 рублей.

Задача решается верно только в том случае, если уравнение к ней составлено правильно. В отличие от обычных уравнений, по которым мы учимся находить корни, уравнения для решения задач имеют своё конкретное применение. Каждый компонент такого уравнения может быть описан в словесной форме. Составляя уравнение, обязательно нужно понимать для чего мы включаем в его состав тот или иной компонент и зачем он нужен.

Также необходимо помнить, что уравнение это равенство, после решения которого левая часть должна будет равняться правой части. Составленное уравнение не должно противоречить этой идее.

Представим, что уравнение это весы с двумя чашами и экраном, показывающим состояние весов.

весы

В данный момент экран показывает знак равенства. Понятно почему левая чаша равна правой чаше — на чашах ничего нет. Состояние весов и отсутствие на чашах чего-либо запишем с помощью следующего равенства:

0 = 0

Положим на левую чашу весов арбуз:

весы арбуз на левой чаше

Левая чаша перевесила правую чашу и экран забил тревогу, показав знак не равно ( ≠ ). Этот знак говорит о том, что левая чаша не равна правой чаше.

Теперь попробуем решить задачу. Пусть требуется узнать сколько весит арбуз, который лежит на левой чаше. Но как это узнать? Ведь наши весы предназначены только для проверки равна ли левая чаша правой.

На помощь приходят уравнения. Вспомним, что уравнение по определению есть равенство, содержащее в себе переменную значение которой требуется найти. Весы в данном случае играют роль этого самого уравнения, а масса арбуза это переменная, значение которой нужно найти. Наша цель правильно составить это уравнение. Понимай, выровнять весы так, чтобы можно было вычислить массу арбуза.

Чтобы выровнять весы, на правую чашу можно положить какой-нибудь тяжелый предмет. Например, положим туда гирю массой 7 кг.

весы арбуз на левой чаше а на правой чаше гиря 7 кг

Теперь наоборот правая чаша перевесила левую. Экран по прежнему показывает, что чаши не равны.

Попробуем на левую чашу положить гирю массой 4 кг

весы арбуз на левой чаше и гиря 4 кг а на правой чаше гиря 7 кг

Теперь весы выровнялись. На рисунке видно, что левая чаша на уровне правой чаши. А экран показывает знак равенства. Этот знак говорит о том, что левая чаша равна правой чаше.

Таким образом мы получили уравнение — равенство, содержащее неизвестное. Левая чаша — это левая часть уравнения, состоящая из компонентов 4 и переменной x (массы арбуза), а правая чаша — это правая часть уравнения, состоящая из компонента 7.

весы арбуз на левой чаше и гиря 4 кг а на правой чаше гиря 7 кг

Ну и нетрудно догадаться, что корень уравнения 4 + x = 7 равен 3. Значит масса арбуза равна 3 кг.

Аналогично дела обстоят и с другими задачами. Чтобы найти какое-нибудь неизвестное значение, к левой или к правой части уравнения добавляют различные элементы: слагаемые, множители, выражения. В школьных задачах эти элементы бывают уже даны. Остается только правильно структурировать их и построить уравнение. Мы же в данном примере занимались подбором, пробуя гири разной массы, чтобы вычислить массу арбуза.

Естественно, те данные которые даны в задаче сначала нужно привести к виду, при котором их можно включить в уравнение. Поэтому, как говорят «хочешь не хочешь, а думать придётся».

Рассмотрим следующую задачу. Возраст отца равен возрасту сына и дочери вместе. Сын вдвое старше дочери и на двадцать лет моложе отца. Сколько лет каждому?

Возраст дочери можно обозначить через x. Если сын вдвое старше дочери, то его возраст будет обозначаться как 2x. В условии задачи сказано, что вместе возраст дочери и сына равен возрасту отца. Значит возраст отца будет обозначаться суммой x + 2x

весы возраст отца и сына и дочери таблица

В выражении x plus 2x можно привести подобные слагаемые. Тогда возраст отца будет обозначаться как 3x

Теперь составим уравнение. Нам нужно получить равенство в котором можно найти неизвестное x. Воспользуемся весами. На левую чашу положим возраст отца (3x), а на правую чашу возраст сына (2x)

весы возраст отца и сына

Понятно почему левая чаша перевесила правую и почему экран показывает знак ( ≠ ). Ведь логично, что возраст отца больше возраста сына.

Но нам нужно уравнять весы, чтобы можно было вычислить неизвестное x. Для этого к правой чаше нужно прибавить какое-нибудь число. Какое именно число указано в задаче. В условии было сказано, что сын моложе отца на 20 лет. Значит 20 лет это то самое число, которое нужно положить на весы.

Весы выровнятся, если мы эти 20 лет добавим на правую чашу весов. Иными словами, вырастим сына до возраста отца

весы возраст отца и сына плюс 20 лет на левой чаше

Теперь весы выровнялись. Получилось уравнение 3x ravno 2x plus 20, которое решается легко:

2x + 20 = 3x решение

В начале решения данной задачи через переменную x мы обозначили возраст дочери. Теперь мы нашли значение этой переменной. Дочери 20 лет.

Далее было сказано, что сын двое старше дочери, значит сыну (20 × 2), то есть 40 лет.

Ну и наконец вычислим возраст отца. В задаче было сказано, что он равен сумме возрастов сына и дочери, то есть (20 + 40) лет.

2x + 20 = 3x решение таблица

Вернемся к середине задачи и обратим внимание на один момент. Когда мы положили на весы возраст отца и возраст сына, левая чаша перевесила правую

весы возраст отца и сына

Но мы решили эту проблему, добавив на правую чашу еще 20 лет. В результате весы выровнялись и мы получили равенство 3x ravno 2x plus 20

Но можно было не добавлять к правой чаше эти 20 лет, а вычесть их из левой. Мы получили бы равенство и в таком случае

весы возраст отца и сына минус 20 лет на другой чаше

В этот раз получается уравнение 3x minus 20 ravno 2x. Корень уравнения по прежнему равен 20

3x minus 20 ravno 2x решение

То есть уравнения 3x ravno 2x plus 20 и 3x minus 20 ravno 2x являются равносильными. А мы помним, что у равносильных уравнений корни совпадают. Если внимательно посмотреть на эти два уравнения, то можно увидеть что второе уравнение получено путем переноса числа 20 из правой части в левую с противоположным знаком. А это действие, как было указано в предыдущем уроке, не меняет корней уравнения.

Также нужно обратить внимание на то, что в начале решения задачи возрасты каждого члена семьи можно было обозначить через другие выражения.

Скажем возраст сына обозначить через x и поскольку он двое старше дочери, то возраст дочери обозначить через x вторых (понимай сделать её младше сына в два раза). А возраст отца поскольку он является суммой возрастов сына и дочери обозначить через выражение x plus x na 2. Ну и напоследок для построения логически правильного уравнения, к возрасту сына нужно  прибавить число 20, ведь отец старше на двадцать лет. В итоге получается совсем другое уравнение x plus 20 ravno x plus x na 2. Решим это уравнение

x plus 20 ravno x plus x na 2 решение

Как видно ответы к задаче не поменялись. Сыну по прежнему 40 лет. Дочери по прежнему сорок вторых лет, а отцу 40 + 20 лет.

Другими словами, задача может решаться различными методами. Поэтому не следует отчаиваться, что не получается решить ту или иную задачу. Но нужно иметь ввиду, что существует наиболее простые пути решения задачи. К центру города можно доехать различными маршрутами, но всегда существует наиболее удобный, быстрый и безопасный маршрут.


Примеры решения задач

Задача 1. В двух пачках всего 30 тетрадей. Если бы из первой пачки переложили во вторую 2 тетради, то в первой пачке стало бы вдвое больше тетрадей, чем во второй. Сколько тетрадей было в каждой пачке?

Решение

Обозначим через x количество тетрадей, которое было в первой пачке. Если всего тетрадей было 30, а переменная x это количество тетрадей из первой пачке, то количество тетрадей во второй пачке будет обозначаться через выражение 30 − x. То есть от общего количества тетрадей вычитаем количество тетрадей из первой пачки и тем самым получаем количество тетрадей из второй пачки.

таблица количество тетрадей в первой и во второй пачке

Далее сказано, что если переложить 2 тетради из первой пачки во вторую, то в первой пачке окажется вдвое больше тетрадей. Итак, снимем с первой пачки две тетради

количество тетрадей в первой и во второй пачке строим уравнение 2

и добавим эти две тетради во вторую пачку

количество тетрадей в первой и во второй пачке строим уравнение 3

Выражения из которых мы будем составлять уравнение теперь принимают следующий вид:

таблица 2 количество тетрадей в первой и во второй пачке

Попробуем составить уравнение из имеющихся выражений. Положим на весы обе пачки тетрадей

весы количество тетрадей в первой и во второй пачке

Левая чаша тяжелее правой. Это потому, что в условии задачи сказано, что после того как из первой пачки взяли две тетради и положили их во вторую, количество тетрадей в первой пачке стало вдвое больше, чем во второй.

Чтобы выровнять весы и получить уравнение, увеличим правую часть вдвое. Для этого умножим её на 2

весы количество тетрадей в первой и во второй пачке рис 2

Получается уравнение x minus 2 ravno 2 na 0 minus x plus 2. Решим данное уравнение:

x minus 2 ravno 2 na 0 minus x plus 2 решение

Первую пачку мы обозначали через переменную x. Теперь мы нашли её значение. Переменная x равна 22. Значит в первой пачке было 22 тетради.

А вторую пачку мы обозначали через выражение 30 − x и поскольку значение переменой x теперь известно, то можно вычислить количество тетрадей во второй пачке. Оно равно 30 − 22, то есть 8 шт.


Задача 2. Два человека чистили картофель. Один очищал в минуту две картофелины, а второй — три картофелины. Вместе они очистили 400 шт. Сколько времени работал каждый, если второй проработал на 25 минут больше первого?

Решение

Обозначим через x время работы первого человека. Поскольку второй человек проработал на 25 минут больше первого, то его время будет обозначаться через выражение x plus 25

Первый рабочий в минуту очищал 2 картофелины, и поскольку он работал x минут, то всего он очистил 2x картофелин.

Второй человек в минуту очищал три картофелины, и поскольку он работал x plus 25 минут, то всего он очистил 3 na x plus 25 картофелин.

Вместе они очистили 400 картофелин

таблица два человека очистили картофелины

Из имеющихся компонентов составим и решим уравнение. В левой части уравнения будут картофелины, очищенные каждым человеком, а в правой части их сумма:

2x plus 3x plus 75 ravno 400 решение

В начале решения данной задачи через переменную x мы обозначили время работы первого человека. Теперь мы нашли значение этой переменной. Первый человек работал 65 минут.

А второй человек работал x plus 25 минут, и поскольку значение переменной x теперь известно, то можно вычислить время работы второго человека — оно равно 65 + 25, то есть 90 мин.


Задача из Учебника по алгебре Андрея Петровича Киселева. Из сортов чая составлена смесь в 32 кг. Килограмм первого сорта стоит 8 руб., а второго сорта 6 руб. 50 коп. Сколько килограммов взято того и другого сорта, если килограмм смеси стоит (без прибыли и убытка) 7 руб. 10 коп.?

Решение

Обозначим через x массу чая первого сорта. Тогда масса чая второго сорта будет обозначаться через выражение 32 − x

таблица 1 масса чая первого и второго сорта

Килограмм чая первого сорта стоит 8 руб. Если эти восемь рублей умножить на количество килограмм чая первого сорта, то можно будет узнать во сколько рублей обошлись x кг чая первого сорта.

А килограмм чая второго сорта стоит 6 руб. 50 коп. Если эти 6 руб. 50 коп. умножить на 32 − x, то можно узнать во сколько рублей обошлись 32 − x кг чая второго сорта.

В условии сказано, что килограмм смеси стоит 7 руб. 10 коп. Всего же было приготовлено 32 кг смеси. Умножим 7 руб. 10 коп. на 32 мы сможем узнать сколько стоит 32 кг смеси.

Выражения из которых мы будем составлять уравнение теперь принимают следующий вид:

таблица 2 стоимость чая первого и второго сорта

Попробуем составить уравнение из имеющихся выражений. Положим на левую чашу весов стоимость смесей чая первого и второго сорта, а на правую чашу положим стоимость 32 кг смеси, то есть общую стоимость смеси, в составе которой оба сорта чая:

весы стоимость чая первого и второго сорта

Получили уравнение 8x plus 650 na 32 minus x ravno 710 na 32. Решим его:

8x plus 650 na 32 minus x ravno 710 na 32 решение

В начале решения данной задачи через переменную x мы обозначили массу чая первого сорта. Теперь мы нашли значение этой переменной. Переменная x равна 12,8. Значит для приготовления смеси было взято 12,8 кг чая первого сорта.

А через выражение 32 − x мы обозначили массу чая второго сорта и поскольку значение переменой x теперь известно, то можно вычислить массу чая второго сорта. Оно равно 32 − 12,8 то есть 19,2. Значит для приготовления смеси было взято 19,2 кг чая второго сорта.


Задача 3. Велосипедист проехал некоторое расстояние со скоростью 8 км/ч. Возвратиться он должен был другой дорогой, которая была на 3 км длиннее первой, и, хотя возвращаясь, ехал со скоростью 9 км/ч, он употребил времени на семь целых одна вторая минут более. Как длинны были дороги?

Решение

Некоторые задачи могут затрагивать темы, которые человек возможно не изучал. Данная задача относится к такому кругу задач. В ней затрагиваются понятия расстояния, скорости и времени. Соответственно, чтобы решить подобную задачу, нужно иметь представление о тех вещах, о которых говорится в задаче. В нашем случае, надо знать что представляет собой расстояние, скорость и время.

В задаче нужно найти расстояния двух дорог. Мы должны составить уравнение, которое позволит вычислить эти расстояния.

Вспомним, как взаимосвязаны расстояние, скорость и время. Каждая из этих величин может быть описана с помощью буквенного уравнения:

расстояние скорость время в картинке

Правую часть одного из этих уравнений мы будем использовать для составления своего уравнения. Чтобы узнать какую именно, нужно вернуться к тексту задачи и обратить внимание на следующий момент:

задача на движение велосипедиста текст на картинке

Следует обратить внимание на момент, где велосипедист на обратном пути употребил времени на семь целых одна вторая минут более. Эта подсказка указывает нам, что можно воспользоваться уравнением t ravno s na v , а именно его правой частью. Это позволит нам составить уравнение, которое содержит переменную S.

Итак, обозначим длину первой дороги через S. Этот путь велосипедист проехал со скоростью 8 км/ч. Время за которое он преодолел этот путь будет обозначаться выражением s na 8, поскольку время это отношение пройденного расстояния к скорости

s na 8 картинка

Обратная дорога для велосипедиста была длиннее на 3 км. Поэтому её расстояние будет обозначаться через выражение + 3. Эту дорогу велосипедист проехал со скоростью 9 км/ч. А значит время за которое он преодолел этот путь будет обозначаться выражением s plus 3 na 9.

s plus 3 na 9 картинка

Теперь составим уравнение из имеющихся выражений

весы два расстояния на чашах

Правая чаша тяжелее левой. Это потому, что в задаче сказано, что на обратную дорогу велосипедист затратил времени на семь целых одна вторая больше.

Чтобы уравнять весы прибавим к левой части эти самые семь целых одна вторая минут. Но сначала переведем минуты в часы, поскольку в задаче скорость измеряется в километрах в час, а не в метрах в минуту.

Чтобы семь целых одна вторая минут перевести в часы, нужно разделить их на 6075 разделить на 60 решение в дробном виде

семь целых одна вторая минут составляют одна восьмая2 часа. Прибавляем эти одна восьмая2 часа к левой части уравнения:

весы два расстояния на чашах равенство

Получается уравнение s na 8 plus 1 na 8 ravno s plus 3 na 9 . Решим данное уравнение. Чтобы избавиться от дробей, обе части части можно умножить на 72. Далее пользуясь известными тождественными преобразованиями, найдем значение переменной S

s na 8 plus 1 na 8 ravno s plus 3 na 9 решение

Через переменную S мы обозначали расстояние первой дороги. Теперь мы нашли значение этой переменной. Переменная S равна 15. Значит расстояние первой дороги составляет 15 км.

А расстояние второй дороги мы обозначили через выражение + 3, и поскольку значение переменной S теперь известно, то можно вычислить расстояние второй дороги. Это расстояние равно сумме 15 + 3, то есть 18 км.


Задача 4. По шоссе идут две машины с одной и той же скоростью. Если первая увеличит скорость на 10 км/ч, а вторая уменьшит скорость на 10 км/ч, то первая за 2 ч пройдет столько же, сколько вторая за 3 ч. С какой скоростью идут автомашины?

Решение

Обозначим через v скорость каждой машины. Далее в задаче приводятся подсказки: скорость первой машины увеличить на 10 км/ч, а скорость второй — уменьшить на 10 км/ч. Воспользуемся этой подсказкой

v plus 10 v minus 10

Далее говорится, что при таких скоростях (увеличенных и уменьшенных на 10 км/ч) первая машина пройдет за 2 часа столько же расстояния сколько вторая за 3 часа. Фразу «столько же» можно понимать как «расстояние, пройденное первой машиной, будет равно расстоянию, пройденному второй машиной».

Расстояние как мы помним, определяется по формуле формула расстояние для вставки в строку. Нас интересует правая часть этого буквенного уравнения — она позволит нам составить уравнение, содержащее переменную v.

Итак, при скорости v + 10 км/ч первая машина пройдет 2(v+10) км, а вторая пройдет 3(v − 10) км. При таком условии машины пройдут одинаковые расстояния, поэтому для получения уравнения достаточно соединить эти два выражения знаком равенства. Тогда получим уравнение 2v plus 20 ravno 3v minus 30. Решим его:

2v plus 20 ravno 3v minus 30 step 1

В условии задачи было сказано, что машины идут с одинаковой скоростью. Мы обозначили эту скорость через переменную v. Теперь мы нашли значение этой переменной. Переменная v равна 50. Значит скорость обеих машин составляла 50 км/ч.


Задача 5. За 9 ч по течению реки теплоход проходит тот же путь, что за 11 ч против течения. Найдите собственную скорость теплохода, если скорость течения реки 2 км/ч.

Решение

Обозначим через v собственную скорость теплохода. Скорость течения реки равна 2 км/ч. По течению реки скорость теплохода будет составлять v + 2 км/ч, а против течения — (v − 2) км/ч.

В условии задачи сказано, что за 9 ч по течению реки теплоход проходит тот же путь, что за 11 ч против течения. Фразу «тот же путь» можно понимать как «расстояние, пройденное теплоходом по течению реки за 9 часов, равно расстоянию, пройденному теплоходом против течения реки за 11 часов». То есть расстояния будут одинаковыми.

Расстояние определяется по формуле формула расстояние для вставки в строку. Воспользуемся правой частью этого буквенного уравнения для составления своего уравнения.

Итак, за 9 часов по течению реки теплоход пройдет 9(v + 2) км, а за 11 часов против течения — 11(v − 2) км. Поскольку оба выражения описывают одно и то же расстояние, приравняем первое выражение ко второму. В результате получим уравнение 9v plus 18 ravno 11v minus 22. Решим его:

9v plus 18 ravno 11v minus 22 решение

Значит собственная скорость теплохода составляет 20 км/ч.

При решении задач полезной привычкой является заранее определить на каком множестве ищется для неё решение.

Допустим, что в задаче требовалось найти время, за которое пешеход преодолеет указанный путь. Мы обозначили время через переменную t, далее составили уравнение, содержащее эту переменную и нашли её значение.

Из практики мы знаем, что время движения объекта может принимать как целые значения, так и дробные, например 2 ч, 1,5 ч, 0,5 ч. Тогда можно сказать, что решение данной задачи ищется на множестве рациональных чисел Q, поскольку каждое из значений 2 ч, 1,5 ч, 0,5 ч может быть представлено в виде дроби.

Поэтому после того, как неизвестную величину обозначили через переменную, полезно указать к какому множеству эта величина принадлежит. В нашем примере время t принадлежит множеству рациональных чисел Q

tQ

Ещё можно ввести ограничение для переменной t, указав что она может принимать только положительные значения. Действительно, если объект затратил на путь определенное время, то это время не может быть отрицательным. Поэтому рядом с выражением ∈ Q укажем, что её значение должно быть больше нуля:

∈ R, t > 0

Если решив уравнение, мы получим отрицательное значение для переменной t, то можно будет сделать вывод, что задача решена неправильно, поскольку это решение не будет удовлетворять условию ∈ Q, > 0.

Ещё пример. Если бы мы решали задачу в которой требовалось найти количество человек для выполнения той или иной работы, то это количество мы обозначили бы через переменную x. В такой задаче решение искалось бы на множестве натуральных чисел

x ∈ N

Действительно, количество человек является целым числом, например 2 человека, 3 человека, 5 человек. Но никак не 1,5 (один целый человек и половина человека) или 2,3 (два целых человека и еще три десятых человека).

Здесь можно было бы указать, что количество человек должно быть больше нуля, но числа входящие во множество натуральных чисел N сами по себе являются положительными и большими нуля. В этом множестве нет отрицательных чисел и числа 0. Поэтому выражение x > 0 можно не писать.


Задача 6. Для ремонта школы прибыла бригада в которой было в 2,5 раза больше маляров, чем плотников. Вскоре прораб включил в бригаду еще четырех маляров, а двух плотников перевел на другой объект. В результате маляров в бригаде оказалось в 4 раза больше чем плотников. Сколько маляров и сколько плотников было в бригаде первоначально

Решение

Обозначим через x плотников, прибывших на ремонт первоначально.

Количество плотников является целым числом, большим нуля. Поэтому укажем, что x принадлежит множество натуральных чисел

N

Маляров было в 2,5 раза больше, чем плотников. Поэтому количество маляров будет обозначаться как 2,5x.

количество плотников x и количество маляров больших в два с половиной раза

Далее говорится, что прораб включил в бригаду еще четырех маляров, а двух плотников перевел на другой объект. Сделаем для своих выражений тоже самое. Уменьшим количество плотников на 2

кол-во плотников уменьшено на 2

А количество маляров увеличим на 4

кол-во плотников увеличено на 4

Теперь количество плотников и маляров будут обозначаться через следующие выражения:

таблица новое количество плотников и маляров

Попробуем составить уравнение из имеющихся выражений:

весы количество плотников и маляров

Правая чаша больше, поскольку после включения в бригаду ещё четырёх маляров, и перемещения двух плотников на другой объект, количество маляров в бригаде оказалось в 4 раза больше чем плотников. Чтобы уравнять весы, нужно левую чашу увеличить в 4 раза:

весы количество плотников и маляров равные чаши

Получили уравнение 4x minus 8 ravno 25x plus 4. Решим его:

4x minus 8 ravno 25x plus 4 решение

Через переменную x было обозначено первоначальное количество плотников. Теперь мы нашли значение этой переменной.  Переменная x равна 8. Значит 8 плотников было в бригаде первоначально.

А количество маляров было обозначено через выражение 2,5x и поскольку значение переменной x теперь известно, то можно вычислить количество маляров — оно равно 2,5 × 8, то есть 20.

Возвращаемся к началу задачи и удостоверяемся, что соблюдается условие ∈ N. Переменная x равна 8, а элементы множества натуральных чисел N это все числа, начинающиеся с 1, 2, 3 и так далее до бесконечности. В это же множество входит число 8, которое мы нашли.

N

Тоже самое можно сказать о количестве маляров. Число 20 принадлежит множеству натуральных чисел:

20  N


Для понимания сути задачи и правильного составления уравнения, вовсе необязательно использовать модель весов с чашами. Можно использовать и другие модели: отрезки, таблицы, схемы. Можно придумать свою модель, которая хорошо описывала бы суть задачи.

Задача 9. Из бидона отлили 30% молока. В результате в нем осталось 14 л. Сколько литров молока было в бидоне первоначально?

Решение

Искомое значение это первоначальное число литров в бидоне. Изобразим число литров в виде линии и подпишем эту линию как X

x литров в бидоне рисунок 1

Сказано, что из бидона отлили 30% молока. Выделим на рисунке приблизительно 30%

x литров в бидоне рисунок 2

Процент по определению есть одна сотая часть чего-то. Если 30% молока отлили, то остальные 70% остались в бидоне. На эти 70% приходятся 14 литров, указанные в задаче. Выделим на рисунке оставшиеся 70%

x литров в бидоне рисунок 3

Теперь можно составить уравнение. Вспомним, как находить процент от числа. Для этого общее количество чего-то делят на 100 и полученный результат умножают на искомое количество процентов. Замечаем, что 14 литров, составляющих 70% можно получить таким же образом: первоначальное число литров X разделить на 100 и полученный результат умножить на 70. Всё это приравнять к числу 14

x na 100 na 70 ravno 14

Или получить более простое уравнение: 70% записать как 0,70, затем умножить на X и приравнять это выражение к 14

x na 100 na 70 ravno 14 в другом виде решение

Значит первоначально в бидоне было 20 литров молока.


Задача 9. Взяли два сплава золота и серебра. В одном количество этих металлов находится в отношении 1 : 9, а в другом 2 : 3. Сколько нужно взять каждого сплава, чтобы получить 15 кг нового сплава, в котором золото и серебро относилось бы как 1 : 4?

Решение

Попробуем сначала узнать сколько золота и серебра будет содержáться в 15 кг нового сплава. В задаче сказано, что содержание этих металлов должно быть в отношении 1 : 4, то есть на одну часть сплава должно приходиться золото, а на четыре части — серебро. Тогда всего частей в сплаве будет 1 + 4 = 5, а масса одной части будет 15 : 5 = 3 кг.

Определим сколько золота будет содержáться в 15 кг сплава. Для этого 3 кг умножим на количество частей золота:

3 кг × 1 = 3 кг

Определим сколько серебра будет содержáться в 15 кг сплава:

3 кг × 4 = 12 кг

Значит сплав массой 15 кг будет содержать 3 кг золота и 12 кг серебра. Теперь вернёмся к исходным сплавам. Использовать нужно каждый из них. Обозначим через x массу первого сплава, а массу второго сплава можно обозначить через 15 − x

табличка x и 15 minus x

Выразим в процентах все отношения, которые даны в задаче и заполним ими следующую таблицу:

таблица три сплава рисунок 1

В первом сплаве золото и серебро находятся в отношении 1 : 9. Тогда всего частей будет 1 + 9 = 10. Из них золота будет 1 на 10 в процентах, а серебра 9 на 10 в процентах.

Перенесём эти данные в таблицу. 10% занесём в первую строку в графу «процент золота в сплаве», 90% также занесём в первую строку графу «процент серебра в сплаве», а в последнюю графу «масса сплава» занесём переменную x, поскольку так мы обозначили массу первого сплава:

таблица три сплава рисунок 2

Аналогично поступаем со вторым сплавом. Золото и серебро в нём находятся в отношении 2 : 3. Тогда всего частей будет 2 + 3 = 5. Из них золота будет 2 на 5 в процентах, а серебра 3 на 5 в процентах.

Перенесём эти данные в таблицу. 40% занесем во вторую строку в графу «процент золота в сплаве», 60% также занесём во вторую строку графу «процент серебра в сплаве», а в последнюю графу «масса сплава» занесём выражение 15 − x, поскольку так мы обозначили массу второго сплава:

таблица три сплава рисунок 3

Заполним последнюю строку. Полученный сплав массой 15 кг будет содержать 3 кг золота, что составляет 3 на 15 в процентах сплава, а серебра будет 12 на 15 в процентах сплава. В последнюю графу записываем массу полученного сплава 15

таблица три сплава рисунок 4

Теперь по данной таблице можно составить уравнения. Вспоминаем задачи на концентрацию, сплавы и смеси. Если мы отдельно сложим золото обоих сплавов и приравняем эту сумму к массе золота полученного сплава, то сможем узнать чему равно значение x.

Далее для удобства проценты будем выражать в десятичной дроби.

В первом сплаве золота было 0,10x, а во втором сплаве золота было 0,40(15 − x). Тогда в полученном сплаве масса золота будет суммой масс золота первого и второго сплавов и эта масса составляет 20% от нового сплава. А 20% от  нового сплава это 3 кг золота, вычисленные нами ранее. В результате получаем уравнение 0,10+ 0.40(15 − x) = 3. Решим это уравнение:

010x plus 040 na 15 - x ravno 3 решение

Изначально через x мы обозначили массу первого сплава. Теперь мы нашли значение этой переменной. Переменная x равна 10. А массу второго сплава мы обозначили через 15 − x, и поскольку значение переменной x теперь известно, то можно вычислить массу второго сплава, она равна 15 − 10 = 5 кг.

Значит для получения нового сплава массой 15 кг в котором золото и серебро относились бы как 1 : 4, нужно взять 10 кг первого и 5 кг второго сплава.

Уравнение можно было составить, воспользовавшись и вторым столбцом получившейся таблицы. Тогда мы получили бы уравнение 0,90+ 0.60(15 − x) = 12. Корень этого уравнения тоже равен 10

090x plus 060 na 15 minus x ravno 12 решение


Задача 10. Имеется руда из двух пластов с содержанием меди в 6% и 11%. Сколько надо взять бедной руды, чтобы получить при смешивании с богатой 20 тонн с содержанием меди 8%?

Решение

Обозначим через x массу бедной руды. Поскольку нужно получить 20 тонн руды, то богатой руды будет взято 20 − x. Поскольку содержание меди в бедной руде составляет 6%, то в x тоннах руды будет содержáться 0,06тонн меди. В богатой руде содержание меди составляет 11%, а в 20 − тоннах богатой руды будет содержáться 0,11(20 − x) тонн меди.

В получившихся 20 тоннах руды содержание меди должно составлять 8%. Значит в 20 тоннах руды меди будет содержáться 20 × 0,08 = 1,6 тонн.

Сложим выражения 0,06x и 0,11(20 − x) и приравняем эту сумму к 1,6. Получим уравнение  0,06x + 0,11(20 − x) = 1,6

006x plus 011 na 20 minus x ravno 16 рисунок

Решим данное уравнение:

006x plus 011 na 20 minus x ravno 16 решение

Значит для получения 20 тонн руды с содержанием меди 8%, нужно взять 12 тонн бедной руды. Богатой же будет взято 20 − 12 = 8 тонн.


Задача 11. Увеличив среднюю скорость с 250 до 300 м/мин спортсменка стала пробегать дистанцию на 1 мин быстрее. Какова длина дистанции?

Решение

Длину дистанции (или расстояние дистанции) можно описать следующим буквенным уравнением:

дистанция формула рисунок к задаче

Воспользуемся правой частью этого уравнения для составления своего уравнения. Изначально спортсменка пробегала дистанцию со скоростью 250 метров в минуту. При такой скорости длина дистанции будет описываться выражением 250t

Затем спортсменка увеличила свою скорость до 300 метров в минуту. При такой скорости длина дистанции будет описываться выражением 300t

Заметим, что длина дистанции это величина постоянная. От того, что спортсменка увеличит скорость или уменьшит её, длина дистанции останется неизменной.

Это позволяет нам приравнять выражение 250t к выражению 300t, поскольку оба выражения описывают длину одной и той же дистанции

250t = 300t

Но в задаче сказано, что при скорости 300 метров в минуту спортсменка стала пробегать дистанцию на 1 минуту быстрее. Другими словами, при скорости 300 метров в минуту, время движения уменьшится на единицу. Поэтому в уравнении 250= 300t в правой части время нужно уменьшить на единицу:

250t ravno 300 t - 1 решение

Получилось простейшее уравнение. Решим его:

250t ravno 300 t - 1 решение полное

При скорости 250 метров в минуту спортсменка пробегает дистанцию за 6 минут. Зная скорость и время, можно определить длину дистанции:

S = 250 × 6 = 1500 м

А при скорости 300 метров в минуту спортсменка пробегает дистанцию за − 1, то есть за 5 минут. Как было сказано ранее длина дистанции не меняется:

= 300 × 5 = 1500 м


Задача 12. Всадник догоняет пешехода, находящегося впереди него на 15 км. Через сколько часов всадник догонит пешехода, если каждый час первый проезжает по 10 км, а второй проходит только по 4 км?

Решение

Данная задача является задачей на движение. Её можно решить, определив скорость сближения и разделив изначальное расстояние между всадником и пешеходом на эту скорость.

Скорость сближения определяется вычитанием меньшей скорости из большей:

10 км/ч − 4 км/ч = 6 км/ч (скорость сближения)

С каждым часом расстояние в 15 километров будут сокращаться на 6 км. Чтобы узнать, когда оно сократится полностью (когда всадник догонит пешехода), нужно 15 разделить на 6

15 : 6 = 2,5 ч

2,5 ч это два целых часа и половина часа. А половина часа это 30 минут. Значит всадник догонит пешехода через 2 часа 30 минут.

всадник догоняет пешехода рисунок 1

Решим эту задачу с помощью уравнения.

Будем считать, что пешеход и всадник вышли в путь из одного и того же места. Пешеход вышел раньше всадника и успел преодолеть 15 км

всадник догоняет пешехода рисунок 2

После этого вслед за ним в путь вышел всадник со скоростью 10 км/ч. А скорость пешехода составляет только 4 км/ч. Это значит, что всадник через некоторое время догонит пешехода. Это время нам нужно найти.

Когда всадник догонит пешехода это будет означать, что они вместе прошли одинаковое расстояние. Расстояние, пройденное всадником и пешеходом описывается следующим уравнением:

дистанция формула рисунок к задаче

Воспользуемся правой частью этого уравнения для составления своего уравнения.

Расстояние, пройденное всадником, будет описываться выражением 10t. Поскольку пешеход вышел в путь раньше всадника и успел преодолеть 15 км, то расстояние пройденное им будет описываться выражением 4t + 15.

На момент, когда всадник догонит пешехода, оба они пройдут одинаковое расстояние. Это позволяет нам приравнять расстояния, пройденные всадником и пешеходом:

10t ravno 4t plus 15

Получилось простейшее уравнение. Решим его:

10t ravno 4t plus 15 решение


Задачи для самостоятельного решения

Задача 1. Из одного города в другой пассажирский поезд приезжает на 45 мин быстрее товарного. Вычисли расстояние между городами, если скорость пассажирского поезда 48 км/ч, а товарного 36 км/ч.

Решение

Скорости поездов в данной задаче измеряются в километрах в час. Поэтому 45 мин, указанные в задаче, переведем в часы. 45 мин это 0,75 ч

Обозначим время, за которое товарный поезд приезжает в город, через переменную t. Поскольку пассажирский поезд приезжает в этот город на 0,75 ч быстрее, то время его движения будет обозначаться через выражение t − 0,75

Пассажирский поезд преодолел 48(t − 0.75) км, а товарный 36t км. Поскольку речь идет об одном и том же расстоянии, приравняем первое выражение ко второму. В результате получим уравнение 48(t − 0.75) = 36t. Решим его:

Теперь вычислим расстояние между городами. Для этого скорость товарного поезда (36 км/ч) умножим на время его движения t. Значение переменной t теперь известно — оно равно трём часам

36 × 3 = 108 км

Для вычисления расстояния можно воспользоваться и скоростью пассажирского поезда. Но в этом случае значение переменной t необходимо уменьшить на 0,75 поскольку пассажирский поезд затратил времени на 0,75 ч меньше

48 × (3 − 0,75) = 144 − 36 = 108 км

Ответ: расстояние между городами равно 108 км.

Задача 2. Из двух городов, расстояние между которыми 150 км, одновременно навстречу друг другу выехали два автомобиля. Скорость одного автомобиля 65 км/ч, а второго 60 км/ч. Через сколько часов они встретились?

Решение

Пусть t время через которое автомобили встретились. Тогда первый автомобиль на момент встречи проедет 65t км, а второй 60t км. Сложим эти расстояния и приравняем к 150. Получим уравнение 65+ 60= 150

Значение переменной t равно 1,2. Значит автомобили встретились через 1,2 часа.

Ответ: автомобили встретились через 1,2 часа.

Задача 3. В трех цехах завода всего 685 рабочих. Во втором цехе рабочих в три раза больше, чем в первом, а в третьем — на 15 рабочих меньше, чем во втором цехе. Сколько рабочих в каждом цехе?

Решение

Пусть x рабочих было в первом цехе. Во втором цехе было в три раза больше, чем в первом, поэтому количество рабочих во втором цехе можно обозначить через выражение 3x. В третьем цехе было на 15 рабочих меньше, чем во втором. Поэтому количество рабочих в третьем цехе можно обозначить через выражение 3x − 15.

В задаче сказано, что всего рабочих было 685. Поэтому можно сложить выражения x, 3x, 3x − 15 и приравнять эту сумму к числу 685. В результате получим уравнение x + 3x + (3x − 15) = 685

Через переменную x было обозначено количество рабочих в первом цехе. Теперь мы нашли значение этой переменной, оно равно 100. Значит в первом цехе было 100 рабочих.

Во втором цехе было 3x рабочих, то есть 3 × 100 = 300. А в третьем цехе было 3x − 15, то есть 3 × 100 − 15 = 285

Ответ: в первом цехе было 100 рабочих, во втором — 300, в третьем — 285.

Задача 4. Две ремонтные мастерские в течение недели должны отремонтировать по плану 18 моторов. Первая мастерская выполнила план на 120%, а вторая — на 125%, поэтому в течение недели отремонтировали 22 мотора. Какой план по ремонту моторов на неделю имела каждая мастерская?

Решение

Пусть x моторов должна была отремонтировать первая мастерская. Тогда вторая мастерская должна была отремонтировать 18 − x моторов.

Поскольку первая мастерская выполнила свой план на 120%, это означает что она отремонтировала 1,2x моторов . А вторая мастерская выполнила свой план на 125%, значит она отремонтировала 1,25(18 − x) моторов.

В задаче сказано, что было отремонтировано 22 мотора. Поэтому можно сложить выражения 1,2x и 1,25(18 − x) , затем приравнять эту сумму к числу 22. В результате получим уравнение 1,2x +  1,25(18 − x22

Через переменную x было обозначено количество моторов, которые должна была отремонтировать первая мастерская. Теперь мы нашли значение этой переменной, она равна 10. Значит первая мастерская должна была отремонтировать 10 моторов.

А через выражение 18 − x было обозначено количество моторов, которые должна была отремонтировать вторая мастерская. Значит вторая мастерская должна была отремонтировать 18 − 10 = 8 моторов.

Ответ: первая мастерская должна была отремонтировать 10 моторов, а вторая — 8 моторов.

Задача 5. Цена товара повысилась на 30% и составляет теперь 91 руб. Сколько стоил товар до повышения цены?

Решение

Пусть x рублей стоил товар до повышения цены. Если цена увеличилась на 30% это означает, что она увеличилась на 0,30x рублей. После повышения цены товар начал стоить 91 руб. Сложим x с 0,30x и приравняем эту сумму к 91. В результате получим уравнение + 0.30= 91

Значит до повышения цены товар стоил 70 рублей.

Ответ: до повышения цены товар стоил 70 рублей.

Задача 6. Число увеличили на 25%. На сколько процентов надо уменьшить новое число, чтобы получилось исходное?

Решение

Пусть x — исходное число. Увеличим его на 25%. Получим выражение x + 0,25x. Приведем подобные слагаемые, получим x + 0,25x = 1.25x.

Узнаем какую часть исходное число x составляет от нового числа 1,25x

Если новое число 1,25x считать за 100%, а исходное число x составляет от него 80%, то уменьшив новое число на 20% можно получить исходное число x

Ответ: чтобы получить исходное число, новое число нужно уменьшить на 20%.

Задача 7. При увеличении числа на 20% получилось 144. Найти первоначальное значение числа.

Решение

Пусть x — первоначальное число. Увеличим его на 20%. Получим выражение x + 0,20x. Приравняем эту сумму к числу 144, получим уравнение x + 0,20= 144

Ответ: первоначальное значение числа равно 120.

Задача 8. При уменьшении числа на 10% получилось 45. Найти первоначальное значение числа.

Решение

Пусть x — первоначальное число. Уменьшим его на 10%. Получим выражение x − 0,10x. Приравняем эту разность к числу 45, получим уравнение x − 0,10x = 45

Ответ: первоначальное значение числа равно 50.

Задача 9. Цена альбома была снижена сначала на 15%, потом еще на 15 руб. Новая цена альбома после двух снижений 19 руб. Определить его первоначальную цену.

Решение

Пусть x рублей — первоначальная цена альбома. Снизим эту цену на 15%, получим x − 0,15x. Снизим цену ещё на 15 руб., получим x − 0,15x − 15. После этих снижений альбом стал стоить 19 руб. Приравняем выражение x − 0,15x − 15 к числу 19, получим уравнение x − 0,15x − 15 = 19

Ответ: первоначальная цена альбома составляет 40 руб.

Задача 10. Трава при сушке теряет 80% своей массы. Сколько тонн травы нужно накосить, чтобы получить 4 т сена?

Решение

Если 80% массы теряется, то на оставшиеся 20% будут приходиться 4 т сена. Пусть x тонн травы требуется для получения 4 т сена. Если 4 т будут составлять 20% травы, то можно составить уравнение:

Ответ: для получения 4 т сена, нужно накосить 20 т травы.

Задача 11. Сколько килограммов 20%-го раствора соли нужно добавить к 1 кг 10%-го раствора, чтобы получить 12%-й раствор соли?

Решение

Пусть x кг 20%-го раствора соли нужно добавить к 1 кг 10%-го раствора.

В 1 кг 10%-го раствора соли содержится 0,1 кг соли. А в x кг 20%-го раствора соли содержится 0,20кг соли.

После добавления x кг 20%-го раствора в новом растворе будет содержáться 0,12(1 + x) кг соли. Сложим выражения 0,1 и 0,20x, затем приравняем эту сумму к выражению 0,12(1 + x). В результате получим уравнение  0,1 + 0,20x = 0,12(1 + x)

Ответ: чтобы получить 12%-й раствор соли, нужно к 1 кг 10%-го раствора добавить 0,25 кг 20%-го раствора.

Задача 12. Даны два раствора соли в воде, концентрации которых равны 20% и 30%. Сколько килограммов каждого раствора нужно смешать в одном сосуде, чтобы получить 25 кг 25,2%-го раствора?

Решение

Пусть x кг первого раствора нужно взять. Поскольку требуется приготовить 25 кг раствора, то массу второго раствора можно обозначить через выражение 25 − x.

В первом растворе будет содержáться 0,20x кг соли, а втором — 0,30(25 − x) кг соли. В полученном растворе содержание соли будет 25 × 0,252 = 6,3 кг. Сложим выражения 0,20x и 0,30(25 − x), затем приравняем эту сумму к 6,3. В результате получим уравнение

Значит первого раствора нужно взять 12 кг, а второго 25 − 12 = 13 кг.

Ответ: первого раствора нужно взять 12 кг, а второго 13 кг.


Понравился урок?
Вступай в нашу новую группу Вконтакте и начни получать уведомления о новых уроках

Возникло желание поддержать проект?
Используй кнопку ниже


Нередко уравнения выручают нас при решении самых разнообразных задач – по математике, физике, механике, экономики и других областей.

Задачи с помощью уравнений 1Рассмотрим общий порядок решения задач с помощью уравнений.

1. Вводим переменные. Иными словами, буквами x, y, z мы обозначаем неизвестные нам величины, которые нам необходимо найти по условию задачи либо которые необходимы для отыскания искомых величин.

2. Составляем уравнение (систему уравнений) – при помощи введенных переменных и данных в условии задачи величин.

3. Решаем составленное уравнение (систему уравнений) и анализируем полученные данные (отбираем из решений те, которые подходят по смыслу задачи).

4. Если буквами x, y, z были обозначены не искомые величины, то при помощи полученных результатов находим ответ на вопрос задачи.

Применим полученные знания на практике и решим задачи.

Задача 1.

Для перевозки 60 т груза понадобилось некоторое количество машин. Из-за ремонта на дороге на каждую машину пришлось грузить на 0,5 т меньше, чем предполагалось, что привело к увеличению общего числа машин на 4 единицы. Какое количество машин было необходимо первоначально?

Решение.

1. Обозначим через х первоначальное количество машин. Тогда всего было использовано (х + 4) машин.

2. Предполагалось, что каждая машина равномерно празделит 60 т груза, т.е. на одну машину будет погружено 60/х т, но фактически на 1 машину было погружено 60/(х + 4) т, что на 0,5 меньше, чем планировалось.

3. На основе введенных переменных и выведенных выражений составим и решим уравнение:

60/х – 60/(х + 4) = 0,5

60/х – 60/(х + 4) = 0,5 ∙ 2х(х + 4)

120(х + 4) – 120х = х(х + 4)

120х + 480 – 120х = х2 + 4х

х2 + 4х – 480 = 0.

По теореме Виетта, х1 + х2 = -4, а х1 ∙ х2 = -480. Значит, х1 = -24, х2 = 20.

4. Анализируем полученные результаты. Число -24 не подходит по смыслу задачи (количество машин не может быть выражено отрицательным числом). Значит, наше решение – х2 = 20, т.е. первоначально понадобилось 20 машин.

Ответ: 20.

Задача 2.

Моторная лодка, двигаясь со скоростью 20 км/ч, прошла расстояние между А и В по реке туда и обратно без остановок за 6 ч 15 мин. Расстояние между А и В составляет 60 км. Найти скорость течения реки.

Решение.

1. Примем за х скорость течения реки.

2. Т.к. по условию задачи лодка двигалась в обоих направлениях (туда и обратно), то по течению она шла со скоростью (20 + х) км/ч, против течения – со скоростью (20 – х) км/ч.

3. Чтобы найти время, нужно расстояние разделить на скорость, т.е. t = s/v. Иными словами, путь по течению займет у лодки 60/(20 + х) ч, а обратный путь – путь против течения займет 60/(20 – х) ч. По условию известно, что весь путь занял 6 ч 15 мин, т.е. 25/4 ч.

4. Используя вышеизложенные сведения, составим и решим уравнение:

60/(20 + х) + 60/(20 – х) = 25/4

60/(20 + х) + 60/(20 – х) = 25/4 ∙ 4(20 + х)(20 – х)

240(20 – х) + 240(20 + х) = 25(20 + х)(20 – х)

4800 – 240х + 4800 + 240х = 25(400 – х2)

9600 = 10000 – 25х2

25 х2 = 10000 – 9600

25 х2 = 400

х2 = 16

х1 = -4, х2 = 4.

5. Анализируем полученные результаты. Число -4 не подходит по смыслу задачи (скорость течения не может быть выражена отрицательным числом). Значит, наше решение – х2 = 4, т.е. скорость течения реки составляет 4 км/ч.

Ответ: 4.

Задача 3.

Есть кусок сплава меди с оловом массой 12 кг, содержащий 45% меди. Сколько чистого олова необходимо прибавить к этому куску , чтобы получившийся новый сплав содержал 40% меди?

Решение.

1. Примем за х массу добавленного олова. Тогда массу получившегося сплава мы обозначим как (12 + х) кг. Этот сплав содержит 40% меди, значит в новом сплаве меди будет 0,4(12 + х).

2. Исходный сплав массой 12 кг содержал 45% меди, значит, в нем было меди 0,45 ∙ 12 кг.Задачи с помощью уравнений 2

3. Т.к. масса меди в исходном и в новом сплаве одинакова, приходим к уравнению:

0,4(12 + х) = 0,45 ∙ 12

4,8 + 0,4х = 5,4

0,4х = 5,4 – 4,8

0,4х = 0,6

х = 0,6 : 0,4

х = 1,5.

Значит, к исходному сплаву необходимо добавить 1,5 кг олова.

Ответ: необходимо 1,5 кг олова.

© blog.tutoronline.ru,
при полном или частичном копировании материала ссылка на первоисточник обязательна.

Решение задач составлением уравнения

Разделы: Математика

«Умственную самодеятельность, сообразительность и смекалку нельзя ни «вдолбить», ни «вложить» ни в чью голову. Результаты надёжны лишь тогда, когда введение в область математических знаний осуществляется в лёгкой и приятной форме, на предметах и примерах обыденной и повседневной обстановки, подобранных с надлежащим остроумием и занимательностью».

Е.И. Игнатьев «В царстве смекалки»

Цель обучающая: Научить анализировать условие задачи, выбирать рациональные способы решения, составлять уравнение, решать его, проверять правильность решения.

Цель воспитательная: Развитие абстрактного и логического мышления.

I. Актуализация опорных знаний.

1) Что называется уравнением? Что называется корнем уравнения? Что значит решить уравнение?

2) Решите уравнение .

Решение: а) ; ; x= ;

б) НОЗ=6; ; ; ; ;

3) Составьте буквенное выражение.

Наташа купила блокнот за m рублей, книгу на 20 рублей дороже блокнота и ручку в 2.5 раза дешевле книги. Сколько рублей стоит ручка?

Решение: (m+20) рублей цена книги, рублей цена ручки.

4) Из двух сёл, расстояние между которыми 10 км навстречу друг другу вышли мальчик и девочка и встретились через 2 часа. Скорость мальчика 3 км/час. Найдите скорость девочки.

Решение: а) 3*2=6(км) прошел мальчик, 10-6=4(км) прошла девочка, 4:2=2(км/ч) скорость девочки

б) 10:2=5(км/ч) скорость сближения, 5-3=2(км/ч) скорость девочки.

II. Сообщение темы и цели урока.

III. Решение задач составлением уравнения.

1) Андрей старше Олега на 4 года, а Олег старше Бориса в 1,5 раза. Вместе им 36 лет. Сколько лет каждому из них?

Первый ряд решает задачу, взяв за неизвестную величину возраст Андрея, второй ряд – возраст Олега, третий ряд-возраст Бориса. А затем каждый ряд объясняет своё решение у доски.

Условие задачи Решение уравнения Проверка
Х лет Андрею,

лет Борису,

16+12+8=36
Х лет Олегу

лет Борису

12+16+8=36
Х лет Борису

(1,5х+4) лет Андрею

8+12+16=36

Если в задаче несколько неизвестных величин, лучше обозначить буквой наименьшую из них.

2) Два пешехода вышли одновременно навстречу друг другу из двух посёлков и встретились через 3ч. Расстояние между посёлками 30 км. Найдите скорость каждого пешехода, если у одного она на 2 км/ч меньше, чем у другого.

а) Ученики решают задачу составлением уравнения.

Х км/ч – скорость 1-го пешехода

(х+2) км/ч – скорость 2-го пешехода

3х км прошел 1-й пешеход

3(х+2) км прошел 2-й пешеход

3х+3(х+2)=30 4
3х+3х+6=30 +6
6х+6=30 +12
6х=24 +18
Х=4 =30

б) Затем решают арифметическим способом.

10-2=8(км/ч)-две скорости 1-го пешехода.

8:2=4(км/ч)-скорость 1-го пешехода.

4+2=6(км/ч)-скорость 2-го пешехода.

Вывод. Арифметическое решение задачи более рациональное

3)Для распечатки 340 страниц были использованы две копировальные машины. Первая машина работала 10 минут, а вторая 15 минут. Сколько страниц в минуту печатает каждая машина, если первая печатает на 4 страницы больше, чем вторая?

Снова дети решают задачу алгебраически и арифметически.

А) х страниц за 1 минуту печатает 2-я машина,

(х+4) страниц за 1 минуту напечатает 1-я машина,

15х страниц напечатает 2 машина,

10(х+4) страниц напечатает 2 машина,

15х+10(х+4=340 12
15х+10х+40=340 +16
25х+40=340 +180
25х=300 +160
Х=12 =340

б) 10*4=40(стр) на столько страниц больше напечатает 1-я машина за 10 минут

340-40=300(стр) напечатали бы обе машины вместе, если бы у них были одинаковые скорости.

10+15=25(мин) работали обе машины.

300:25=12(стр) за 1-у минуту печатает 2-я машина.

12+4=16 (стр) за 1-у минуту печатает 1-я машина.

При решении задач с помощью уравнения поступают следующим образом:

  1. Внимательно читают условие задачи.
  2. Обозначают неизвестную величину буквой.
  3. Переводят условие задачи на алгебраический язык.
  4. Составляют уравнение.
  5. Решают уравнение.
  6. Проверяют правильность решения

V. Домашнее задание.

Решить задачу алгебраически и арифметически.

Охотничья собака спугнула зайца, который сидел под кустом в 150м от неё. Через сколько минут собака догонит зайца, если она пробегает за 6 минут 3,6км, а заяц только 3км?

  1. Л.В. Кузнецова, Е.А. Бунимович, Б.П. Пигарев, С.Б. Суворова «Алгебра» сборник заданий для проведения письменного экзамена по алгебре за курс основной школы. Москва. «Дрофа». 2001г; стр 43, работа №31, вариант 1(5); стр 50, работа №38, вариант 2(7); стр 65, работа №48, вариант 1(6).
  2. Составители Т.А. Братусь, Н.А. Жарковская, Е.А Рисс, Т.Е Савелова “Детский математический календарь 2001-2002”. Санкт – Питербург стр10.
  3. Н.Т. Кострикина “Задачи повышенной трудности в курсе алгебры 7-9 классов”. Москва. “Просвещение” 1991 стр5-19.

Решение задач с помощью уравнений

Тема урока: § 6. Решение задач с помощью уравнений. Приведены все необходимые и достаточные сведения для решения текстовых задач с помощью составления уравнений.

Введение

В школьной математике есть целый кладезь текстовых задач, которые решаются универсальным методом построения уравнения (модели) исходя из условия.

Сам факт того, что огромное количество самых разнообразных задач поддаются решению с помощью составления линейного уравнения, говорит нам, что метод решений является действительно универсальным.

Обычно условия задач удается перевести на математический язык. Полученное уравнение – это следствие перевода нашего условия с русского языка на язык алгебры. Зачастую фактической стороной повествования задачи является описание реальной ситуации, какого либо процесса, события.

Чтобы получить ответ – уравнение нужно решить, полученный корень уравнения будет являться решением, разумеется необходимо еще проверить, не является ли результат противоречивым относительно условия.

Алгоритм решения текстовых задач с помощью уравнений

Для решения задачи с помощью уравнения делают следующие действия:

  1. Обозначают некоторое неизвестное буквой и, пользуясь условием, составляют уравнение.
  2. Решают уравнение.
  3. Истолковывают результат.

Примеры решений

Задача 1.
В мешке было в 3 раза меньше монет, чем в сундуке. После того как из мешка переложили 24 монеты, в сундуке их стало в 7 раз больше, чем в мешке. Сколько было монет в мешке и сколько в сундуке?

Пусть $x$ – количество монет в мешке, а значит в сундуке: $3x$ монет. После того, как из мешка переложили $24$ монеты, в сундуке стало: $3x+24$, а в мешке $x-24$. И если в сундуке их стало в $7$ раз больше чем в мешке, то имеем: $3x+24=7(x-24)$.

Ну вот мы и составили уравнение (математическую модель), осталось решить уравнение относительно $x$ и записать ответ.

Решим полученное уравнение: $3x+24=7(x-24)$. Легко увидеть, что уравнение является линейным (узнать как решаются линейные уравнения можно тут.)

Раскроем скобки в правой части уравнения: $3x+24=7x-7cdot 24$. Перенесём все слагаемые содержащие переменную в правую часть, а всё что не содержит $x$ в левую, получим: $24+7cdot 24=7x-3x$. После упрощения получили $192=4x$, разделим обе части уравнения на коэффициент при неизвестном, т.е на $4$, тогда получим $x=48$.

Осталось истолковать ответ.
За переменную $x$ мы обозначали количество монет в мешке, значит в сундуке в три раза больше т.е $3x$.

Монет в мешке: $48$

Монет в сундуке: $48cdot 3=144$

Задача 2.
Купили 3600 кг муки и высыпали её в три мешка. В первый мешок муки вошло в 3 раза больше, чем во второй, а в третий мешок насыпали 800 кг муки. Сколько муки насыпали в первый и сколько во второй мешок?

Пусть в первый мешок насыпали $3x$ кг муки, тогда во второй мешок насыпали $x$ кг. Если сложим количество кг в каждом мешке, то получим $3600$ кг муки. Имеем: $3x+x+800=3600$, решим уравнение классическим методом.

Все слагаемые содержащие $x$ оставим слева, а всё остальное перенесём в правую часть равенства: $3x+x=3600-800$, упростим обе части; $4x=2800$ поделим обе части равенства на $4$ и получим ответ: $x=700$.

Ответ.
За переменную $x$ мы обозначали количество муки во втором мешке, по условию в первом в три раза больше.

Муки в первом мешке: $700cdot 3=2100$ кг.

Муки во втором мешке: $700$ кг.

Задача 3.
В первом мешке в 4 раза больше картофеля, чем во втором. После того, как из одного мешка взяли 40 кг картофеля, а во второй насыпали ещё 5 кг, в обоих мешках картофеля стало поровну. Сколько килограммов картофеля было во втором мешке.

Пусть во втором мешке $x$ кг картофеля, тогда в первом мешке $4x$ кг. Из первого взяли $40$ кг, тогда в первом стало: $4x-40$. Во второй мешок насыпали $5$ кг и теперь в нём: $x+5$ кг картошки. Нам известно, что после этих изменений количество картофеля в мешках стало поровну, запишем это с помощью линейного уравнения:

Решим это линейное уравнение. Все слагаемые содержащие переменную перенесём влево, а свободные члены вправо и получим:

Избавимся от коэффициента при неизвестном и получим ответ:

Ответ.
За переменную $x$ мы обозначали количество кг картошки во втором мешке, по условию в первом в четыре раза больше.

Картошки в первом мешке: $15cdot 4=60$ кг.

Картошки во втором мешке: $15$ кг.

Задача 4.
По шоссе едут две машины с одной и той же скоростью. Если первая увеличит скорость на 20 км/ч, а вторая уменьшит скорость на 20 км/ч, то первая за 2 часа пройдёт то же самое расстояние, что и вторая за 4 часа. Найдите первоначальную скорость машин.

Пусть машины едут со скоростью $v$ км/ч, тогда после ускорения первой машины её скорость стала: $v+20$ км/ч, а скорость второй машины после замедления стала: $v-20$ км/ч. Нам известно по условию, что после изменения скоростей машин, первая проходит за два часа ровно столько, сколько вторая за четыре, тогда имеем:

По известной нам формуле $S=vt$ ($S$ – расстояние, $v$ – скорость, $t$ – время)

Сократим обе части равенства на $2$, тогда получим: $v+20=2(v-20)$. Раскроем скобки в правой части уравнения и сгруппируем все переменные в правой части равенства.

Ответ.
В качестве неизвестной величины в задаче мы взяли $v$ (первоначальную скорость машин).

Первоначальная скорость машин: $v=60$ км/ч.

Задача 5.
В первую бригаду привезли раствора цемента на 50 кг меньше, чем во вторую. Каждый час работы первая бригада расходовала 150 кг раствора, а вторая – 200кг. Через 3 ч работы в первой бригаде осталось раствора в 1,5 раза больше, чем во второй. Сколько раствора привезли в каждую бригаду?

Пусть во вторую бригаду привезли $x$ кг раствора цемента, тогда в первую бригаду привезли $x-50$ кг. Через 3 часа работы у первой бригады осталось $x-50-3cdot 150$ кг цемента, а у второй $x-3cdot 200$ кг.

По условию известно, что через 3 часа работы в первой бригаде осталось в 1,5 раза больше цемента, чем во второй, тогда имеем:

$$x-50-3cdot 150=1,5(x-3cdot 200)$$

Осталось решить данное уравнение относительно $x$ и истолковать ответ.

Упростим и раскроем скобки в правой части, тогда получим:

Если вам неудобно работать с десятичными дробями, то вы всегда можете их переводить в рациональный вид: $1,5=frac<15><10>=frac<3><2>$.

Запишем с учётом перевода дробей и упростим:

Перенесём слагаемые содержащие переменную в правую сторону, а всё остальное в левую:

Домножим обе части на 2 и получим ответ:

Ответ.
В качестве переменной в задаче мы взяли $x$ (кол-во кг цемента который привезли во вторую бригаду), по условию в первую привезли на 50 кг меньше, а значит $x-50$

Кол-во цемента в первой бригаде: $800-50=750$ кг.

Кол-во цемента во второй бригаде: $800$ кг.

Задачи для самостоятельного решения

По контракту работникам причитается 48 франков за каждый отработанный день, а за каждый неотработанный день с них вычитается по 12 франков. Через 30 дней выяснилось, что работникам ничего не причитается. Сколько дней они отработали в течение этих 30 дней?

Пусть работники отработали $n$ дней, тогда $30-n$ дней они не отработали.

В итоге мы понимаем, что за $n$ рабочих дней они зарабатывают $48n$ франков и с них вычитается за $30-n$ не отработанных дней по $12(30-n)$ франков. Тогда ясно, что: $48n-12(30-n)=0$

Ответ: Рабочие отработали 6 дней.

Кирпич весит фунт и полкирпича. Сколько фунтов весит кирпич?

Пусть целый кирпич весит весит $k$ фунтов, тогда имеем:

1 фунт и половина кирпича = целый кирпич.

Бутылка с пробкой стоит 10 копеек, причем бутылка на 9 копеек дороже пробки. Сколько стоит бутылка без пробки?

Пусть бутылка стоит $b$ копеек, а пробка $p$ копеек, тогда:

$b+p=10$ и $b=p+9$, подставив значение $b$ в первое равенство – получим:

Т.е пробка стоит пол копейки, тогда бутылка $9,5$ копеек.

Ответ: 9,5 копеек стоит бутыка без пробки.

На свитер, шапку и шарф израсходовали 555 г шерсти, причем на шапку ушло в 5 раз меньше шерсти, чем на свитер, и на 5 г больше, чем на шарф. Сколько шерсти израсходовали на каждое изделие?

Пусть на свитер потратили $5x$ г шерсти, тогда на шапку ушло $x$ г и на шарф потребовалось $x-5$ г, имеем:

Ответ: На шапку ушло $80$ г, на свитер $5cdot 80=400$ г, на шарф $80-5=75$ г.

Три пионерских звена собрали для школьной библиотеки 65 книг. Первое звено собрало на 10 книг меньше, чем второе, а третье — 30% того числа книг, которое собрали первое и второе звено вместе. Сколько книг собрало каждое звено?

Пусть второе звено собрало $x$ книг, тогда первое собрало $x-10$ книг, а третье $0,3(2x-10)$, имеем:

$$2x-10+0,3cdot 2x-0,3cdot 10=65$$

$$2x+0,3cdot 2x=65+10+0,3cdot 10$$

Ответ: Первое звено собрало $30-10=20$ книг, второе $30$ книг, третье $0,3(60-10)=15$ книг.

Решение задач с помощью уравнений

Решение задачи обычно свóдится к тому, чтобы путем логических рассуждений и вычислений найти значение какой-нибудь величины. Например, найти скорость, время, расстояние, массу какого-нибудь предмета или количество чего-то.

Такую задачу можно решить с помощью уравнения. Для этого искомое значение обозначают через переменную, затем путем логических рассуждений составляют и решают уравнение. Решив уравнение, производят проверку на то, удовлетворяет ли решение уравнения условиям задачи.

Запись выражений, содержащих неизвестное

Решение задачи сопровождается составлением уравнения к этой задаче. На начальном этапе изучения задач желательно научиться составлять буквенные выражения, описывающие ту или иную жизненную ситуацию. Этот этап не является сложным и его можно изучать в процессе решения самой задачи.

Рассмотрим несколько ситуаций, которые можно записать с помощью математического выражения.

Задача 1. Возраст отца x лет. Мама на два года младше. Сын младше отца в 3 раза. Запишите возраст каждого с помощью выражений.

Решение:

Задача 2. Возраст отца x лет, мама на 2 года младше отца. Сын младше отца в 3 раза, дочь младше матери в 3 раза. Запишите возраст каждого с помощью выражений.

Решение:

Задача 3. Возраст отца x лет, мама на 3 года младше отца. Сын младше отца в 3 раза, дочь младше матери в 3 раза. Сколько лет каждому, если общий возраст отца, мамы, сына и дочери составляет 92 года?

Решение:

В данной задаче помимо записи выражений, необходимо вычислить возраст каждого члена семьи.

Сначала запишем возраст каждого члена семьи с помощью выражений. За переменную x примем возраст отца, и далее пользуясь этой переменной составим остальные выражения:

Теперь определим возраст каждого члена семьи. Для этого нам нужно составить и решить уравнение. Все компоненты уравнения у нас уже готовы. Осталось только собрать их воедино.

Общий возраст в 92 года получился путем сложения возрастов папы, мамы, сына и дочери:

Для каждого возраста мы составили математическое выражение. Эти выражения и будут компонентами нашего уравнения. Давайте соберем наше уравнение согласно данной схеме и таблице, которая была приведена выше. То есть слова папа, мама, сын, дочь заменим на соответствующее им в таблице выражение:

Выражение, отвечающее за возраст мамы x − 3, для наглядности было взято в скобки.

Теперь решим получившееся уравнение. Для начала можно раскрыть скобки там, где это можно:

Чтобы освободить уравнение от дробей, умножим обе части на 3

Решим получившееся уравнение, пользуясь известными тождественными преобразованиями:

Мы нашли значение переменной x . Эта переменная отвечала за возраст отца. Значит возраст отца составляет 36 лет.

Зная возраст отца, можно вычислить возрасты остальных членов семьи. Для этого нужно подставить значение переменной x в те выражения, которые отвечают за возраст конкретного члена семьи.

В задаче было сказано, что мама на 3 года младше отца. Ее возраст мы обозначили через выражение x−3. Значение переменной x теперь известно, и чтобы вычислить возраст мамы, нужно в выражении x − 3 вместо x подставить найденное значение 36

x − 3 = 36 − 3 = 33 года маме.

Аналогично определяется возраст остальных членов семьи:

Проверка:

Задача 4. Килограмм яблок стоит x рублей. Запишите выражение, вычисляющее сколько килограмм яблок можно купить на 300 рублей.

Решение

Если килограмм яблок стоит x рублей, то на 300 рублей можно купить килограмм яблок.

Пример. Килограмм яблок стоит 50 рублей. Тогда на 300 рублей можно купить , то есть 6 килограмм яблок.

Задача 5. На x рублей было куплено 5 кг яблок. Запишите выражение, вычисляющее сколько рублей стоит один килограмм яблок.

Решение

Если за 5 кг яблок было уплачено x рублей, то один килограмм будет стоит рублей

Пример. За 300 рублей было куплено 5 кг яблок. Тогда один килограмм яблок будет стоит , то есть 60 рублей.

Задача 6. Том, Джон и Лео на перемене пошли в столовую и купили по бутерброду и по кружке кофе. Бутерброд стоит x рублей, а кружка кофе — 15 рублей. Определите стоимость бутерброда, если известно, что за всё было уплачено 120 рублей?

Решение

Конечно, данная задача проста как три копейки и ее можно решить не прибегая к уравнению. Для этого из 120 рублей нужно вычесть стоимость трех кружек кофе (15 × 3) , и полученный результат разделить на 3

Но наша цель — составить уравнение к задаче и решить это уравнение. Итак, стоимость бутерброда x рублей. Куплено их всего три. Значит увеличив стоимость в три раза, мы получим выражение описывающее сколько рублей было уплачено за три бутерброда

3x — стоимость трех бутербродов

А стоимость трех кружек кофе можно записать как 15 × 3 . 15 это стоимость одной кружки кофе, а 3 множитель (Том, Джон и Лео), увеличивающий эту стоимость в три раза.

По условию задачи за все уплачено 120 рублей. У нас уже появляется примерная схема, что нужно делать:

Выражения, описывающие стоимость трех бутербродов и трех кружек кофе, у нас уже готовы. Это выражения 3x и 15 × 3 . Пользуясь схемой составим уравнение и решим его:

Итак, стоимость одного бутерброда составляет 25 рублей.

Задача решается верно только в том случае, если уравнение к ней составлено правильно. В отличие от обычных уравнений, по которым мы учимся находить корни, уравнения для решения задач имеют своё конкретное применение. Каждый компонент такого уравнения может быть описан в словесной форме. Составляя уравнение, обязательно нужно понимать для чего мы включаем в его состав тот или иной компонент и зачем он нужен.

Также необходимо помнить, что уравнение это равенство, после решения которого левая часть должна будет равняться правой части. Составленное уравнение не должно противоречить этой идее.

Представим, что уравнение это весы с двумя чашами и экраном, показывающим состояние весов.

В данный момент экран показывает знак равенства. Понятно почему левая чаша равна правой чаше — на чашах ничего нет. Состояние весов и отсутствие на чашах чего-либо запишем с помощью следующего равенства:

Положим на левую чашу весов арбуз:

Левая чаша перевесила правую чашу и экран забил тревогу, показав знак не равно ( ≠ ). Этот знак говорит о том, что левая чаша не равна правой чаше.

Теперь попробуем решить задачу. Пусть требуется узнать сколько весит арбуз, который лежит на левой чаше. Но как это узнать? Ведь наши весы предназначены только для проверки равна ли левая чаша правой.

На помощь приходят уравнения. Вспомним, что уравнение по определению есть равенство, содержащее в себе переменную значение которой требуется найти. Весы в данном случае играют роль этого самого уравнения, а масса арбуза это переменная, значение которой нужно найти. Наша цель правильно составить это уравнение. Понимай, выровнять весы так, чтобы можно было вычислить массу арбуза.

Чтобы выровнять весы, на правую чашу можно положить какой-нибудь тяжелый предмет. Например, положим туда гирю массой 7 кг.

Теперь наоборот правая чаша перевесила левую. Экран по прежнему показывает, что чаши не равны.

Попробуем на левую чашу положить гирю массой 4 кг

Теперь весы выровнялись. На рисунке видно, что левая чаша на уровне правой чаши. А экран показывает знак равенства. Этот знак говорит о том, что левая чаша равна правой чаше.

Таким образом мы получили уравнение — равенство, содержащее неизвестное. Левая чаша — это левая часть уравнения, состоящая из компонентов 4 и переменной x (массы арбуза), а правая чаша — это правая часть уравнения, состоящая из компонента 7.

Ну и нетрудно догадаться, что корень уравнения 4 + x = 7 равен 3. Значит масса арбуза равна 3 кг.

Аналогично дела обстоят и с другими задачами. Чтобы найти какое-нибудь неизвестное значение, к левой или к правой части уравнения добавляют различные элементы: слагаемые, множители, выражения. В школьных задачах эти элементы бывают уже даны. Остается только правильно структурировать их и построить уравнение. Мы же в данном примере занимались подбором, пробуя гири разной массы, чтобы вычислить массу арбуза.

Естественно, те данные которые даны в задаче сначала нужно привести к виду, при котором их можно включить в уравнение. Поэтому, как говорят «хочешь не хочешь, а думать придётся».

Рассмотрим следующую задачу. Возраст отца равен возрасту сына и дочери вместе. Сын вдвое старше дочери и на двадцать лет моложе отца. Сколько лет каждому?

Возраст дочери можно обозначить через x . Если сын вдвое старше дочери, то его возраст будет обозначаться как 2x . В условии задачи сказано, что вместе возраст дочери и сына равен возрасту отца. Значит возраст отца будет обозначаться суммой x + 2x

В выражении можно привести подобные слагаемые. Тогда возраст отца будет обозначаться как 3x

Теперь составим уравнение. Нам нужно получить равенство в котором можно найти неизвестное x . Воспользуемся весами. На левую чашу положим возраст отца (3x) , а на правую чашу возраст сына (2x)

Понятно почему левая чаша перевесила правую и почему экран показывает знак ( ≠ ) . Ведь логично, что возраст отца больше возраста сына.

Но нам нужно уравнять весы, чтобы можно было вычислить неизвестное x . Для этого к правой чаше нужно прибавить какое-нибудь число. Какое именно число указано в задаче. В условии было сказано, что сын моложе отца на 20 лет. Значит 20 лет это то самое число, которое нужно положить на весы.

Весы выровнятся, если мы эти 20 лет добавим на правую чашу весов. Иными словами, вырастим сына до возраста отца

Теперь весы выровнялись. Получилось уравнение , которое решается легко:

В начале решения данной задачи через переменную x мы обозначили возраст дочери. Теперь мы нашли значение этой переменной. Дочери 20 лет.

Далее было сказано, что сын двое старше дочери, значит сыну (20 × 2) , то есть 40 лет.

Ну и наконец вычислим возраст отца. В задаче было сказано, что он равен сумме возрастов сына и дочери, то есть (20 + 40) лет.

Вернемся к середине задачи и обратим внимание на один момент. Когда мы положили на весы возраст отца и возраст сына, левая чаша перевесила правую

Но мы решили эту проблему, добавив на правую чашу еще 20 лет. В результате весы выровнялись и мы получили равенство

Но можно было не добавлять к правой чаше эти 20 лет, а вычесть их из левой. Мы получили бы равенство и в таком случае

В этот раз получается уравнение . Корень уравнения по прежнему равен 20

То есть уравнения и являются равносильными. А мы помним, что у равносильных уравнений корни совпадают. Если внимательно посмотреть на эти два уравнения, то можно увидеть что второе уравнение получено путем переноса числа 20 из правой части в левую с противоположным знаком. А это действие, как было указано в предыдущем уроке, не меняет корней уравнения.

Также нужно обратить внимание на то, что в начале решения задачи возрасты каждого члена семьи можно было обозначить через другие выражения.

Скажем возраст сына обозначить через x и поскольку он двое старше дочери, то возраст дочери обозначить через (понимай сделать её младше сына в два раза). А возраст отца поскольку он является суммой возрастов сына и дочери обозначить через выражение . Ну и напоследок для построения логически правильного уравнения, к возрасту сына нужно прибавить число 20, ведь отец старше на двадцать лет. В итоге получается совсем другое уравнение . Решим это уравнение

Как видно ответы к задаче не поменялись. Сыну по прежнему 40 лет. Дочери по прежнему лет, а отцу 40 + 20 лет.

Другими словами, задача может решаться различными методами. Поэтому не следует отчаиваться, что не получается решить ту или иную задачу. Но нужно иметь ввиду, что существует наиболее простые пути решения задачи. К центру города можно доехать различными маршрутами, но всегда существует наиболее удобный, быстрый и безопасный маршрут.

Примеры решения задач

Задача 1. В двух пачках всего 30 тетрадей. Если бы из первой пачки переложили во вторую 2 тетради, то в первой пачке стало бы вдвое больше тетрадей, чем во второй. Сколько тетрадей было в каждой пачке?

Решение

Обозначим через x количество тетрадей, которое было в первой пачке. Если всего тетрадей было 30, а переменная x это количество тетрадей из первой пачке, то количество тетрадей во второй пачке будет обозначаться через выражение 30 − x . То есть от общего количества тетрадей вычитаем количество тетрадей из первой пачки и тем самым получаем количество тетрадей из второй пачки.

Далее сказано, что если переложить 2 тетради из первой пачки во вторую, то в первой пачке окажется вдвое больше тетрадей. Итак, снимем с первой пачки две тетради

и добавим эти две тетради во вторую пачку

Выражения из которых мы будем составлять уравнение теперь принимают следующий вид:

Попробуем составить уравнение из имеющихся выражений. Положим на весы обе пачки тетрадей

Левая чаша тяжелее правой. Это потому, что в условии задачи сказано, что после того как из первой пачки взяли две тетради и положили их во вторую, количество тетрадей в первой пачке стало вдвое больше, чем во второй.

Чтобы выровнять весы и получить уравнение, увеличим правую часть вдвое. Для этого умножим её на 2

Получается уравнение . Решим данное уравнение:

Первую пачку мы обозначали через переменную x . Теперь мы нашли её значение. Переменная x равна 22. Значит в первой пачке было 22 тетради.

А вторую пачку мы обозначали через выражение 30 − x и поскольку значение переменой x теперь известно, то можно вычислить количество тетрадей во второй пачке. Оно равно 30 − 22 , то есть 8 шт .

Задача 2. Два человека чистили картофель. Один очищал в минуту две картофелины, а второй — три картофелины. Вместе они очистили 400 шт. Сколько времени работал каждый, если второй проработал на 25 минут больше первого?

Решение

Обозначим через x время работы первого человека. Поскольку второй человек проработал на 25 минут больше первого, то его время будет обозначаться через выражение

Первый рабочий в минуту очищал 2 картофелины, и поскольку он работал x минут, то всего он очистил 2x картофелин.

Второй человек в минуту очищал три картофелины, и поскольку он работал минут, то всего он очистил картофелин.

Вместе они очистили 400 картофелин

Из имеющихся компонентов составим и решим уравнение. В левой части уравнения будут картофелины, очищенные каждым человеком, а в правой части их сумма:

В начале решения данной задачи через переменную x мы обозначили время работы первого человека. Теперь мы нашли значение этой переменной. Первый человек работал 65 минут.

А второй человек работал минут, и поскольку значение переменной x теперь известно, то можно вычислить время работы второго человека — оно равно 65 + 25 , то есть 90 мин .

Задача из Учебника по алгебре Андрея Петровича Киселева. Из сортов чая составлена смесь в 32 кг. Килограмм первого сорта стоит 8 руб., а второго сорта 6 руб. 50 коп. Сколько килограммов взято того и другого сорта, если килограмм смеси стоит (без прибыли и убытка) 7 руб. 10 коп.?

Решение

Обозначим через x массу чая первого сорта. Тогда масса чая второго сорта будет обозначаться через выражение 32 − x

Килограмм чая первого сорта стоит 8 руб. Если эти восемь рублей умножить на количество килограмм чая первого сорта, то можно будет узнать во сколько рублей обошлись x кг чая первого сорта.

А килограмм чая второго сорта стоит 6 руб. 50 коп. Если эти 6 руб. 50 коп. умножить на 32 − x , то можно узнать во сколько рублей обошлись 32 − x кг чая второго сорта.

В условии сказано, что килограмм смеси стоит 7 руб. 10 коп. Всего же было приготовлено 32 кг смеси. Умножим 7 руб. 10 коп. на 32 мы сможем узнать сколько стоит 32 кг смеси.

Выражения из которых мы будем составлять уравнение теперь принимают следующий вид:

Попробуем составить уравнение из имеющихся выражений. Положим на левую чашу весов стоимость смесей чая первого и второго сорта, а на правую чашу положим стоимость 32 кг смеси, то есть общую стоимость смеси, в составе которой оба сорта чая:

Получили уравнение . Решим его:

В начале решения данной задачи через переменную x мы обозначили массу чая первого сорта. Теперь мы нашли значение этой переменной. Переменная x равна 12,8. Значит для приготовления смеси было взято 12,8 кг чая первого сорта.

А через выражение 32 − x мы обозначили массу чая второго сорта и поскольку значение переменой x теперь известно, то можно вычислить массу чая второго сорта. Оно равно 32 − 12,8 то есть 19,2 . Значит для приготовления смеси было взято 19,2 кг чая второго сорта.

Задача 3. Велосипедист проехал некоторое расстояние со скоростью 8 км/ч. Возвратиться он должен был другой дорогой, которая была на 3 км длиннее первой, и, хотя возвращаясь, ехал со скоростью 9 км/ч, он употребил времени на минут более. Как длинны были дороги?

Решение

Некоторые задачи могут затрагивать темы, которые человек возможно не изучал. Данная задача относится к такому кругу задач. В ней затрагиваются понятия расстояния, скорости и времени. Соответственно, чтобы решить подобную задачу, нужно иметь представление о тех вещах, о которых говорится в задаче. В нашем случае, надо знать что представляет собой расстояние, скорость и время.

В задаче нужно найти расстояния двух дорог. Мы должны составить уравнение, которое позволит вычислить эти расстояния.

Вспомним, как взаимосвязаны расстояние, скорость и время. Каждая из этих величин может быть описана с помощью буквенного уравнения:

Правую часть одного из этих уравнений мы будем использовать для составления своего уравнения. Чтобы узнать какую именно, нужно вернуться к тексту задачи и обратить внимание на следующий момент:

Следует обратить внимание на момент, где велосипедист на обратном пути употребил времени на минут более. Эта подсказка указывает нам, что можно воспользоваться уравнением , а именно его правой частью. Это позволит нам составить уравнение, которое содержит переменную S .

Итак, обозначим длину первой дороги через S . Этот путь велосипедист проехал со скоростью 8 км/ч . Время за которое он преодолел этот путь будет обозначаться выражением , поскольку время это отношение пройденного расстояния к скорости

Обратная дорога для велосипедиста была длиннее на 3 км . Поэтому её расстояние будет обозначаться через выражение S + 3 . Эту дорогу велосипедист проехал со скоростью 9 км/ч . А значит время за которое он преодолел этот путь будет обозначаться выражением .

Теперь составим уравнение из имеющихся выражений

Правая чаша тяжелее левой. Это потому, что в задаче сказано, что на обратную дорогу велосипедист затратил времени на больше.

Чтобы уравнять весы прибавим к левой части эти самые минут. Но сначала переведем минуты в часы, поскольку в задаче скорость измеряется в километрах в час, а не в метрах в минуту.

Чтобы минут перевести в часы, нужно разделить их на 60

минут составляют часа. Прибавляем эти часа к левой части уравнения:

Получается уравнение . Решим данное уравнение. Чтобы избавиться от дробей, обе части части можно умножить на 72. Далее пользуясь известными тождественными преобразованиями, найдем значение переменной S

Через переменную S мы обозначали расстояние первой дороги. Теперь мы нашли значение этой переменной. Переменная S равна 15. Значит расстояние первой дороги составляет 15 км.

А расстояние второй дороги мы обозначили через выражение S + 3 , и поскольку значение переменной S теперь известно, то можно вычислить расстояние второй дороги. Это расстояние равно сумме 15 + 3 , то есть 18 км .

Задача 4. По шоссе идут две машины с одной и той же скоростью. Если первая увеличит скорость на 10 км/ч, а вторая уменьшит скорость на 10 км/ч, то первая за 2 ч пройдет столько же, сколько вторая за 3 ч. С какой скоростью идут автомашины?

Решение

Обозначим через v скорость каждой машины. Далее в задаче приводятся подсказки: скорость первой машины увеличить на 10 км/ч, а скорость второй — уменьшить на 10 км/ч. Воспользуемся этой подсказкой

Далее говорится, что при таких скоростях (увеличенных и уменьшенных на 10 км/ч) первая машина пройдет за 2 часа столько же расстояния сколько вторая за 3 часа. Фразу «столько же» можно понимать как «расстояние, пройденное первой машиной, будет равно расстоянию, пройденному второй машиной».

Расстояние как мы помним, определяется по формуле . Нас интересует правая часть этого буквенного уравнения — она позволит нам составить уравнение, содержащее переменную v .

Итак, при скорости v + 10 км/ч первая машина пройдет 2(v+10) км , а вторая пройдет 3(v − 10) км . При таком условии машины пройдут одинаковые расстояния, поэтому для получения уравнения достаточно соединить эти два выражения знаком равенства. Тогда получим уравнение . Решим его:

В условии задачи было сказано, что машины идут с одинаковой скоростью. Мы обозначили эту скорость через переменную v . Теперь мы нашли значение этой переменной. Переменная v равна 50. Значит скорость обеих машин составляла 50 км/ч.

Задача 5. За 9 ч по течению реки теплоход проходит тот же путь, что за 11 ч против течения. Найдите собственную скорость теплохода, если скорость течения реки 2 км/ч.

Решение

Обозначим через v собственную скорость теплохода. Скорость течения реки равна 2 км/ч. По течению реки скорость теплохода будет составлять v + 2 км/ч , а против течения — (v − 2) км/ч .

В условии задачи сказано, что за 9 ч по течению реки теплоход проходит тот же путь, что за 11 ч против течения. Фразу «тот же путь» можно понимать как «расстояние, пройденное теплоходом по течению реки за 9 часов, равно расстоянию, пройденному теплоходом против течения реки за 11 часов». То есть расстояния будут одинаковыми.

Расстояние определяется по формуле . Воспользуемся правой частью этого буквенного уравнения для составления своего уравнения.

Итак, за 9 часов по течению реки теплоход пройдет 9(v + 2) км , а за 11 часов против течения — 11(v − 2) км . Поскольку оба выражения описывают одно и то же расстояние, приравняем первое выражение ко второму. В результате получим уравнение . Решим его:

Значит собственная скорость теплохода составляет 20 км/ч.

При решении задач полезной привычкой является заранее определить на каком множестве ищется для неё решение.

Допустим, что в задаче требовалось найти время, за которое пешеход преодолеет указанный путь. Мы обозначили время через переменную t , далее составили уравнение, содержащее эту переменную и нашли её значение.

Из практики мы знаем, что время движения объекта может принимать как целые значения, так и дробные, например 2 ч, 1,5 ч, 0,5 ч. Тогда можно сказать, что решение данной задачи ищется на множестве рациональных чисел Q, поскольку каждое из значений 2 ч, 1,5 ч, 0,5 ч может быть представлено в виде дроби.

Поэтому после того, как неизвестную величину обозначили через переменную, полезно указать к какому множеству эта величина принадлежит. В нашем примере время t принадлежит множеству рациональных чисел Q

Ещё можно ввести ограничение для переменной t , указав что она может принимать только положительные значения. Действительно, если объект затратил на путь определенное время, то это время не может быть отрицательным. Поэтому рядом с выражением tQ укажем, что её значение должно быть больше нуля:

Если решив уравнение, мы получим отрицательное значение для переменной t , то можно будет сделать вывод, что задача решена неправильно, поскольку это решение не будет удовлетворять условию tQ , t > 0 .

Ещё пример. Если бы мы решали задачу в которой требовалось найти количество человек для выполнения той или иной работы, то это количество мы обозначили бы через переменную x . В такой задаче решение искалось бы на множестве натуральных чисел

Действительно, количество человек является целым числом, например 2 человека, 3 человека, 5 человек. Но никак не 1,5 (один целый человек и половина человека) или 2,3 (два целых человека и еще три десятых человека).

Здесь можно было бы указать, что количество человек должно быть больше нуля, но числа входящие во множество натуральных чисел N сами по себе являются положительными и большими нуля. В этом множестве нет отрицательных чисел и числа 0. Поэтому выражение x > 0 можно не писать.

Задача 6. Для ремонта школы прибыла бригада в которой было в 2,5 раза больше маляров, чем плотников. Вскоре прораб включил в бригаду еще четырех маляров, а двух плотников перевел на другой объект. В результате маляров в бригаде оказалось в 4 раза больше чем плотников. Сколько маляров и сколько плотников было в бригаде первоначально

Решение

Обозначим через x плотников, прибывших на ремонт первоначально.

Количество плотников является целым числом, большим нуля. Поэтому укажем, что x принадлежит множество натуральных чисел

Маляров было в 2,5 раза больше, чем плотников. Поэтому количество маляров будет обозначаться как 2,5x .

Далее говорится, что прораб включил в бригаду еще четырех маляров, а двух плотников перевел на другой объект. Сделаем для своих выражений тоже самое. Уменьшим количество плотников на 2

А количество маляров увеличим на 4

Теперь количество плотников и маляров будут обозначаться через следующие выражения:

Попробуем составить уравнение из имеющихся выражений:

Правая чаша больше, поскольку после включения в бригаду ещё четырёх маляров, и перемещения двух плотников на другой объект, количество маляров в бригаде оказалось в 4 раза больше чем плотников. Чтобы уравнять весы, нужно левую чашу увеличить в 4 раза:

Получили уравнение . Решим его:

Через переменную x было обозначено первоначальное количество плотников. Теперь мы нашли значение этой переменной. Переменная x равна 8. Значит 8 плотников было в бригаде первоначально.

А количество маляров было обозначено через выражение 2,5 x и поскольку значение переменной x теперь известно, то можно вычислить количество маляров — оно равно 2,5 × 8 , то есть 20 .

Возвращаемся к началу задачи и удостоверяемся, что соблюдается условие xN. Переменная x равна 8, а элементы множества натуральных чисел N это все числа, начинающиеся с 1, 2, 3 и так далее до бесконечности. В это же множество входит число 8, которое мы нашли.

Тоже самое можно сказать о количестве маляров. Число 20 принадлежит множеству натуральных чисел:

Для понимания сути задачи и правильного составления уравнения, вовсе необязательно использовать модель весов с чашами. Можно использовать и другие модели: отрезки, таблицы, схемы. Можно придумать свою модель, которая хорошо описывала бы суть задачи.

Задача 9. Из бидона отлили 30% молока. В результате в нем осталось 14 л. Сколько литров молока было в бидоне первоначально?

Решение

Искомое значение это первоначальное число литров в бидоне. Изобразим число литров в виде линии и подпишем эту линию как X

Сказано, что из бидона отлили 30% молока. Выделим на рисунке приблизительно 30%

Процент по определению есть одна сотая часть чего-то. Если 30% молока отлили, то остальные 70% остались в бидоне. На эти 70% приходятся 14 литров, указанные в задаче. Выделим на рисунке оставшиеся 70%

Теперь можно составить уравнение. Вспомним, как находить процент от числа. Для этого общее количество чего-то делят на 100 и полученный результат умножают на искомое количество процентов. Замечаем, что 14 литров, составляющих 70% можно получить таким же образом: первоначальное число литров X разделить на 100 и полученный результат умножить на 70. Всё это приравнять к числу 14

Или получить более простое уравнение: 70% записать как 0,70, затем умножить на X и приравнять это выражение к 14

Значит первоначально в бидоне было 20 литров молока.

Задача 9. Взяли два сплава золота и серебра. В одном количество этих металлов находится в отношении 1 : 9, а в другом 2 : 3. Сколько нужно взять каждого сплава, чтобы получить 15 кг нового сплава, в котором золото и серебро относилось бы как 1 : 4?

Решение

Попробуем сначала узнать сколько золота и серебра будет содержáться в 15 кг нового сплава. В задаче сказано, что содержание этих металлов должно быть в отношении 1 : 4, то есть на одну часть сплава должно приходиться золото, а на четыре части — серебро. Тогда всего частей в сплаве будет 1 + 4 = 5, а масса одной части будет 15 : 5 = 3 кг.

Определим сколько золота будет содержáться в 15 кг сплава. Для этого 3 кг умножим на количество частей золота:

Определим сколько серебра будет содержáться в 15 кг сплава:

Значит сплав массой 15 кг будет содержать 3 кг золота и 12 кг серебра. Теперь вернёмся к исходным сплавам. Использовать нужно каждый из них. Обозначим через x массу первого сплава, а массу второго сплава можно обозначить через 15 − x

Выразим в процентах все отношения, которые даны в задаче и заполним ими следующую таблицу:

В первом сплаве золото и серебро находятся в отношении 1 : 9. Тогда всего частей будет 1 + 9 = 10 . Из них золота будет , а серебра .

Перенесём эти данные в таблицу. 10% занесём в первую строку в графу «процент золота в сплаве», 90% также занесём в первую строку графу «процент серебра в сплаве», а в последнюю графу «масса сплава» занесём переменную x , поскольку так мы обозначили массу первого сплава:

Аналогично поступаем со вторым сплавом. Золото и серебро в нём находятся в отношении 2 : 3. Тогда всего частей будет 2 + 3 = 5. Из них золота будет , а серебра .

Перенесём эти данные в таблицу. 40% занесем во вторую строку в графу «процент золота в сплаве», 60% также занесём во вторую строку графу «процент серебра в сплаве», а в последнюю графу «масса сплава» занесём выражение 15 − x , поскольку так мы обозначили массу второго сплава:

Заполним последнюю строку. Полученный сплав массой 15 кг будет содержать 3 кг золота, что составляет сплава, а серебра будет сплава. В последнюю графу записываем массу полученного сплава 15

Теперь по данной таблице можно составить уравнения. Вспоминаем задачи на концентрацию, сплавы и смеси. Если мы отдельно сложим золото обоих сплавов и приравняем эту сумму к массе золота полученного сплава, то сможем узнать чему равно значение x.

Далее для удобства проценты будем выражать в десятичной дроби.

В первом сплаве золота было 0,10x , а во втором сплаве золота было 0,40(15 − x) . Тогда в полученном сплаве масса золота будет суммой масс золота первого и второго сплавов и эта масса составляет 20% от нового сплава. А 20% от нового сплава это 3 кг золота, вычисленные нами ранее. В результате получаем уравнение 0,10x + 0.40(15 − x) = 3 . Решим это уравнение:

Изначально через x мы обозначили массу первого сплава. Теперь мы нашли значение этой переменной. Переменная x равна 10. А массу второго сплава мы обозначили через 15 − x , и поскольку значение переменной x теперь известно, то можно вычислить массу второго сплава, она равна 15 − 10 = 5 кг .

Значит для получения нового сплава массой 15 кг в котором золото и серебро относились бы как 1 : 4, нужно взять 10 кг первого и 5 кг второго сплава.

Уравнение можно было составить, воспользовавшись и вторым столбцом получившейся таблицы. Тогда мы получили бы уравнение 0,90x + 0.60(15 − x) = 12. Корень этого уравнения тоже равен 10

Задача 10. Имеется руда из двух пластов с содержанием меди в 6% и 11%. Сколько надо взять бедной руды, чтобы получить при смешивании с богатой 20 тонн с содержанием меди 8%?

Решение

Обозначим через x массу бедной руды. Поскольку нужно получить 20 тонн руды, то богатой руды будет взято 20 − x . Поскольку содержание меди в бедной руде составляет 6%, то в x тоннах руды будет содержáться 0,06x тонн меди. В богатой руде содержание меди составляет 11%, а в 20 − x тоннах богатой руды будет содержáться 0,11(20 − x) тонн меди.

В получившихся 20 тоннах руды содержание меди должно составлять 8%. Значит в 20 тоннах руды меди будет содержáться 20 × 0,08 = 1,6 тонн.

Сложим выражения 0,06x и 0,11(20 − x) и приравняем эту сумму к 1,6. Получим уравнение 0,06x + 0,11(20 − x) = 1,6

Решим данное уравнение:

Значит для получения 20 тонн руды с содержанием меди 8%, нужно взять 12 тонн бедной руды. Богатой же будет взято 20 − 12 = 8 тонн.

Задача 11. Увеличив среднюю скорость с 250 до 300 м/мин спортсменка стала пробегать дистанцию на 1 мин быстрее. Какова длина дистанции?

Решение

Длину дистанции (или расстояние дистанции) можно описать следующим буквенным уравнением:

Воспользуемся правой частью этого уравнения для составления своего уравнения. Изначально спортсменка пробегала дистанцию со скоростью 250 метров в минуту. При такой скорости длина дистанции будет описываться выражением 250t

Затем спортсменка увеличила свою скорость до 300 метров в минуту. При такой скорости длина дистанции будет описываться выражением 300t

Заметим, что длина дистанции это величина постоянная. От того, что спортсменка увеличит скорость или уменьшит её, длина дистанции останется неизменной.

Это позволяет нам приравнять выражение 250t к выражению 300t , поскольку оба выражения описывают длину одной и той же дистанции

Но в задаче сказано, что при скорости 300 метров в минуту спортсменка стала пробегать дистанцию на 1 минуту быстрее. Другими словами, при скорости 300 метров в минуту, время движения уменьшится на единицу. Поэтому в уравнении 250t = 300t в правой части время нужно уменьшить на единицу:

Получилось простейшее уравнение. Решим его:

При скорости 250 метров в минуту спортсменка пробегает дистанцию за 6 минут. Зная скорость и время, можно определить длину дистанции:

S = 250 × 6 = 1500 м

А при скорости 300 метров в минуту спортсменка пробегает дистанцию за t − 1 , то есть за 5 минут. Как было сказано ранее длина дистанции не меняется:

S = 300 × 5 = 1500 м

Задача 12. Всадник догоняет пешехода, находящегося впереди него на 15 км. Через сколько часов всадник догонит пешехода, если каждый час первый проезжает по 10 км, а второй проходит только по 4 км?

Решение

Данная задача является задачей на движение. Её можно решить, определив скорость сближения и разделив изначальное расстояние между всадником и пешеходом на эту скорость.

Скорость сближения определяется вычитанием меньшей скорости из большей:

10 км/ч − 4 км/ч = 6 км/ч (скорость сближения)

С каждым часом расстояние в 15 километров будут сокращаться на 6 км. Чтобы узнать, когда оно сократится полностью (когда всадник догонит пешехода), нужно 15 разделить на 6

2,5 ч это два целых часа и половина часа. А половина часа это 30 минут. Значит всадник догонит пешехода через 2 часа 30 минут.

Решим эту задачу с помощью уравнения.

Будем считать, что пешеход и всадник вышли в путь из одного и того же места. Пешеход вышел раньше всадника и успел преодолеть 15 км

После этого вслед за ним в путь вышел всадник со скоростью 10 км/ч. А скорость пешехода составляет только 4 км/ч. Это значит, что всадник через некоторое время догонит пешехода. Это время нам нужно найти.

Когда всадник догонит пешехода это будет означать, что они вместе прошли одинаковое расстояние. Расстояние, пройденное всадником и пешеходом описывается следующим уравнением:

Воспользуемся правой частью этого уравнения для составления своего уравнения.

Расстояние, пройденное всадником, будет описываться выражением 10t . Поскольку пешеход вышел в путь раньше всадника и успел преодолеть 15 км, то расстояние пройденное им будет описываться выражением 4t + 15 .

На момент, когда всадник догонит пешехода, оба они пройдут одинаковое расстояние. Это позволяет нам приравнять расстояния, пройденные всадником и пешеходом:

Получилось простейшее уравнение. Решим его:

Задачи для самостоятельного решения

Решение

Скорости поездов в данной задаче измеряются в километрах в час. Поэтому 45 мин, указанные в задаче, переведем в часы. 45 мин это 0,75 ч

Обозначим время, за которое товарный поезд приезжает в город, через переменную t . Поскольку пассажирский поезд приезжает в этот город на 0,75 ч быстрее, то время его движения будет обозначаться через выражение t − 0,75

Пассажирский поезд преодолел 48(t − 0.75) км, а товарный 36t км. Поскольку речь идет об одном и том же расстоянии, приравняем первое выражение ко второму. В результате получим уравнение 48(t − 0.75) = 36t . Решим его:

Теперь вычислим расстояние между городами. Для этого скорость товарного поезда (36 км/ч) умножим на время его движения t. Значение переменной t теперь известно — оно равно трём часам

Для вычисления расстояния можно воспользоваться и скоростью пассажирского поезда. Но в этом случае значение переменной t необходимо уменьшить на 0,75 поскольку пассажирский поезд затратил времени на 0,75 ч меньше

48 × (3 − 0,75) = 144 − 36 = 108 км

Ответ: расстояние между городами равно 108 км.

Решение

Пусть t время через которое автомобили встретились. Тогда первый автомобиль на момент встречи проедет 65t км, а второй 60t км. Сложим эти расстояния и приравняем к 150. Получим уравнение 65t + 60t = 150

Значение переменной t равно 1,2. Значит автомобили встретились через 1,2 часа.

Ответ: автомобили встретились через 1,2 часа.

Решение

Пусть x рабочих было в первом цехе. Во втором цехе было в три раза больше, чем в первом, поэтому количество рабочих во втором цехе можно обозначить через выражение 3x . В третьем цехе было на 15 рабочих меньше, чем во втором. Поэтому количество рабочих в третьем цехе можно обозначить через выражение 3x − 15 .

В задаче сказано, что всего рабочих было 685. Поэтому можно сложить выражения x, 3x, 3x − 15 и приравнять эту сумму к числу 685. В результате получим уравнение x + 3x + (3x − 15) = 685

Через переменную x было обозначено количество рабочих в первом цехе. Теперь мы нашли значение этой переменной, оно равно 100. Значит в первом цехе было 100 рабочих.

Во втором цехе было 3x рабочих, то есть 3 × 100 = 300 . А в третьем цехе было 3x − 15 , то есть 3 × 100 − 15 = 285

Ответ: в первом цехе было 100 рабочих, во втором — 300, в третьем — 285.

Решение

Пусть x моторов должна была отремонтировать первая мастерская. Тогда вторая мастерская должна была отремонтировать 18 − x моторов .

Поскольку первая мастерская выполнила свой план на 120%, это означает что она отремонтировала 1,2x моторов . А вторая мастерская выполнила свой план на 125%, значит она отремонтировала 1,25(18 − x) моторов.

В задаче сказано, что было отремонтировано 22 мотора. Поэтому можно сложить выражения 1,2x и 1,25(18 − x) , затем приравнять эту сумму к числу 22. В результате получим уравнение 1,2x + 1,25(18 − x) = 22

Через переменную x было обозначено количество моторов, которые должна была отремонтировать первая мастерская. Теперь мы нашли значение этой переменной, она равна 10. Значит первая мастерская должна была отремонтировать 10 моторов.

А через выражение 18 − x было обозначено количество моторов, которые должна была отремонтировать вторая мастерская. Значит вторая мастерская должна была отремонтировать 18 − 10 = 8 моторов.

Ответ: первая мастерская должна была отремонтировать 10 моторов, а вторая — 8 моторов.

Решение

Пусть x рублей стоил товар до повышения цены. Если цена увеличилась на 30% это означает, что она увеличилась на 0,30x рублей. После повышения цены товар начал стоить 91 руб. Сложим x с 0,30x и приравняем эту сумму к 91. В результате получим уравнение x + 0.30x = 91

Значит до повышения цены товар стоил 70 рублей.

Ответ: до повышения цены товар стоил 70 рублей.

Решение

Пусть x — исходное число. Увеличим его на 25%. Получим выражение x + 0,25x . Приведем подобные слагаемые, получим x + 0,25x = 1.25x .

Узнаем какую часть исходное число x составляет от нового числа 1,25x

Если новое число 1,25x считать за 100%, а исходное число x составляет от него 80%, то уменьшив новое число на 20% можно получить исходное число x

Ответ: чтобы получить исходное число, новое число нужно уменьшить на 20%.

Решение

Пусть x — первоначальное число. Увеличим его на 20%. Получим выражение x + 0,20x . Приравняем эту сумму к числу 144, получим уравнение x + 0,20x = 144

Ответ: первоначальное значение числа равно 120.

Решение

Пусть x — первоначальное число. Уменьшим его на 10%. Получим выражение x − 0,10x . Приравняем эту разность к числу 45, получим уравнение x − 0,10x = 45

Ответ: первоначальное значение числа равно 50.

Решение

Пусть x рублей — первоначальная цена альбома. Снизим эту цену на 15%, получим x − 0,15x . Снизим цену ещё на 15 руб., получим x − 0,15x − 15 . После этих снижений альбом стал стоить 19 руб. Приравняем выражение x − 0,15x − 15 к числу 19, получим уравнение x − 0,15x − 15 = 19

Ответ: первоначальная цена альбома составляет 40 руб.

Решение

Если 80% массы теряется, то на оставшиеся 20% будут приходиться 4 т сена. Пусть x тонн травы требуется для получения 4 т сена. Если 4 т будут составлять 20% травы, то можно составить уравнение:

Ответ: для получения 4 т сена, нужно накосить 20 т травы.

Решение

Пусть x кг 20%-го раствора соли нужно добавить к 1 кг 10%-го раствора.

В 1 кг 10%-го раствора соли содержится 0,1 кг соли. А в x кг 20%-го раствора соли содержится 0,20 x кг соли.

После добавления x кг 20%-го раствора в новом растворе будет содержáться 0,12(1 + x) кг соли. Сложим выражения 0,1 и 0,20x , затем приравняем эту сумму к выражению 0,12(1 + x) . В результате получим уравнение 0,1 + 0,20x = 0,12(1 + x)

Ответ: чтобы получить 12%-й раствор соли, нужно к 1 кг 10%-го раствора добавить 0,25 кг 20%-го раствора.

Решение

Пусть x кг первого раствора нужно взять. Поскольку требуется приготовить 25 кг раствора, то массу второго раствора можно обозначить через выражение 25 − x.

В первом растворе будет содержáться 0,20x кг соли, а втором — 0,30(25 − x) кг соли. В полученном растворе содержание соли будет 25 × 0,252 = 6,3 кг. Сложим выражения 0,20x и 0,30(25 − x), затем приравняем эту сумму к 6,3. В результате получим уравнение

Значит первого раствора нужно взять 12 кг, а второго 25 − 12 = 13 кг.

Ответ: первого раствора нужно взять 12 кг, а второго 13 кг.

Понравился урок?
Вступай в нашу новую группу Вконтакте и начни получать уведомления о новых уроках

Возникло желание поддержать проект?
Используй кнопку ниже

44 thoughts on “Решение задач с помощью уравнений”

Вау новый урок. Я рад что вернулись)) После работы обязательно буду учить этот урок.

не смог решить ни одной задачи из примеров решения…

[spoiler title=”источники:”]

http://reshu.su/algebra/06/

[/spoiler]

Умение решать уравнения необходимо для того, чтобы решать какие-то практические задачи по математике, физике, механике, экономике и другим предметам.

Пример:

в одном баке воды было в (4) раза больше, чем в другом. Из первого бака перелили в другой (36) литров и воды в баках стало поровну. Сколько литров воды было в каждом баке?

Решение:

сначала введём переменную, с помощью которой обозначим неизвестную нам величину, которую необходимо найти по условию задачи.

Пусть (x) л — количество воды, которое было до переливания во втором баке.

Тогда в первом баке её было (4x) л.

После переливания в первом баке осталось ((4x) (– 36)) л воды, а во втором стало ((x + 36)) л.

По условию задачи известно, что после переливания в обоих баках воды стало поровну. Составим уравнение:

(4x) (– 36 = x + 36).

Эту часть рассуждений при решении задач называют составлением математической модели.

На этом этапе текст задачи переводится с обычного языка на математический язык.

Математической моделью является составленное уравнение.

Затем начинается второй этап, называемый работой с математической моделью.

Здесь решается составленное уравнение:

4x−36=x+36;4x−x=36+36;3x=72;x=24.

Решив уравнение, переходим к третьему этапуответу на вопрос задачи.

Решив уравнение, получили (x=24), а за (x) принято количество воды в литрах, которое было до переливания во втором баке.

Значит, во втором баке было (24) л воды. По условию задачи в первом баке было в (4) раза больше воды, чем во втором. Значит, в первом баке было:

(24·4=96) (л).

Ответ: в одном баке было (24) л воды, а в другом баке было (96) л воды.

Таким образом, в ходе решения было выделено три этапа математического моделирования:

1) составление математической модели (составление уравнения по условию задачи);

2) работа с математической моделью (решение уравнения);

3) ответ на вопрос задачи.

Для составления математической модели нужно провести анализ задачи, результаты которого можно оформить в виде таблицы, схемы, рисунка, краткой записи.

Добавить комментарий